neurosensory

Ace your homework & exams now with Quizwiz!

During a class on stroke, a junior nursing student asks what the clinical manifestations of stroke are. What would be the instructor's best answer?

"Clinical manifestations of a stroke depend on the area of the cortex, the affected hemisphere, the degree of blockage, and the availability of collateral circulation." Clinical manifestations following a stroke are highly variable and depend on the area of the cerebral cortex and the affected hemisphere, the degree of blockage (total, partial), and the presence or absence of adequate collateral circulation. (Collateral circulation is circulation formed by smaller blood vessels branching off from or near larger occluded vessels.) Clinical manifestations of a stroke do not depend on the cardiovascular health of the client or how quickly the clot can be dissolved. Clinical manifestations of a stroke are not "general" but individual.

A client with Guillain-Barré syndrome has paralysis affecting the respiratory muscles and requires mechanical ventilation. When the client asks the nurse about the paralysis, how should the nurse respond?

"The paralysis caused by this disease is temporary." The nurse should inform the client that the paralysis that accompanies Guillain-Barré syndrome is only temporary. Return of motor function begins proximally and extends distally in the legs.

Aneurysm rebleeding occurs most frequently during which time frame after the initial hemorrhage?

12

A client weighing 132 lb is brought to the emergency department in status epilepticus. The physician asks the nurse to prepare diazepam 0.25 mg/kg. How many milligrams will be given to this client?

15 Step 1: 2.2 lb / 1 kg = 132 lb / X kg 132 lb = 2.2 X 60 kg = X Step 2: 1 kg / 0.25 mg = 60 kg / X mg 15 mg = X

Bell palsy is a disorder of which cranial nerve?

7th (facial)

deaf culture

A community of people who share the same values, beliefs, experiences, and practices regarding deafness; may include hearing people who are children or close friends of Deaf individuals; native language is American Sign Language (ASL)

A client with quadriplegia is in spinal shock. What finding should the nurse expect?

Absence of reflexes along with flaccid extremities During the period immediately following a spinal cord injury, spinal shock occurs. In spinal shock, all reflexes are absent and the extremities are flaccid. When spinal shock subsides, the client will demonstrate positive Babinski's reflex, hyperreflexia, and spasticity of all four extremities.

The nurse is caring for an 82-year-old client diagnosed with cranial arteritis. What is the priority nursing intervention?

Administer corticosteroids as ordered. Cranial arteritis is caused by inflammation, which can lead to visual impairment or rupture of the vessel. Administering the corticosteroid as ordered can decrease the chance of losing vision or vessel rupture. The client should receive an analgesic (acetaminophen) for the pain, but the corticosteroid should help decrease the pain and prevent complications. The nurse should assess for weight loss, but that can be determined after the medication is administered. Signs and symptoms of inflammation should be documented by the nurse after measures have been taken to decrease complications.

A client who recently experienced a stroke tells the nurse that he has double vision. Which nursing intervention is most appropriate?

Alternatively patch one eye every 2 hours. Patching one eye at a time relieves diplopia (double vision). Closing the eyes and making the room dark aren't the most appropriate options because they deprive the client of sensory input. Artificial tears relieve eye dryness but don't treat diplopia.

A nurse is caring for a client who has returned to his room after a carotid endarterectomy. Which action should the nurse take first?

Ask the client if he has trouble breathing. the nurse should first assess the client's breathing. A complication of a carotid endarterectomy is an incisional hematoma, which could compress the trachea causing breathing difficulty for the client. Although the other measures are important actions, they aren't the nurse's top priority.

Which positions is used to help reduce intracranial pressure (ICP)?

Avoiding flexion of the neck with use of a cervical collar Use of a cervical collar promotes venous drainage and prevents jugular vein distortion, which can increase ICP. Slight elevation of the head is maintained to aid in venous drainage unless otherwise prescribed. Extreme rotation of the neck is avoided because compression or distortion of the jugular veins increases ICP. Extreme hip flexion is avoided because this position causes an increase in intra-abdominal pressure and intrathoracic pressure, which can produce a rise in ICP.

The nurse obtains a Snellen eye chart when assessing cranial nerve function. Which cranial nerve is the nurse testing when using the chart?

CN 2 The nurse assesses vision and thus the optic nerve (cranial nerve II) by use of a Snellen eye chart.

There are 12 pairs of cranial nerves. Only three are sensory. Select the cranial nerve that is affected with decreased visual fields.

CN 2 The three sensory cranial nerves are I, II and VIII. Cranial nerve II (optic) is affected with decreased visual fields and acuity.

The nurse is performing a neurologic assessment on a client diagnosed with a stroke and cannot elicit a gag reflex. This deficit is related to which of the following cranial nerves?

CN X CN X is the vagus nerve and has to do with the gag reflex, laryngeal hoarseness, swallowing ability, and the symmetrical rise of the uvula and soft palate. CN VII is the facial nerve and has to do with symmetry of facial movements and the ability to discriminate between the tastes of sugar and salt. The inability to close one eyelid indicates impairment of this nerve. CN VIII is the acoustic nerve. It has to do with hearing, air and bone conduction, and balance. CN III is the oculomotor nerve and has to do with pupillary response, conjugate movements, and nystagmus.

A patient presents to the emergency room with complaints of having an "exploding headache" for the last 2 hours. The patient is immediately seen by a triage nurse who suspects the patient is experiencing a stroke. Which of the following is a possible cause based on the characteristic symptom?

Cerebral aneurysm A cerebral aneurysm is a type of hemorrhagic stroke that is characterized by an exploding headache.

Which is the primary medical management of arthropod-borne virus (arboviral) encephalitis?

Controlling seizures and increased intracranial pressure There is no specific medication for arboviral encephalitis; therefore symptom management is key. Medical management is aimed at controlling seizures and increased intracranial pressure.

A client is waiting in a triage area to learn the medical status of family members following a motor vehicle accident. The client is pacing, taking deep breaths, and handwringing. Considering the effects in the body systems, what does the nurse anticipate the liver will do?

Convert glycogen to glucose for immediate use. When the body is under stress, the sympathetic nervous system is activated readying the body for action. The effect of the body is to mobilize stored glycogen to glucose to provide additional energy for body action.

After the patient has received tPA, the nurse knows to check vital signs every 30 minutes for 6 hours. Which of the following readings would require calling the provider?

Diastolic pressure of 110 mm Hg A diastolic pressure reading of over 105 mm Hg warrants notifying the health care provider. The other choices are within normal range.

Which is a sympathetic effect of the nervous system?

Dilated pupils Dilated pupils are a sympathetic effect of the nervous system, whereas constricted pupils are a parasympathetic effect. Decreased blood pressure is a parasympathetic effect, whereas increased blood pressure is a sympathetic effect. Increased peristalsis is a parasympathetic effect, but decreased peristalsis is a sympathetic effect. Decreased respiratory rate is a parasympathetic effect, and increased respiratory rate is a sympathetic effect.

The nurse is evaluating the transmission of a report from a paramedic unit to the emergency department. The medic reports that a client is unconscious with edema of the head and face and Battle sign. What clinical picture would the nurse anticipate?

Edema to the head and bruising over the mastoid process Battle sign is the presence of bruising of the mastoid process behind the ear. It is not related to periorbital bleeding, lacerations, or fixed pupils.

A client with a traumatic brain injury has developed increased intracranial pressure resulting in diabetes insipidus. While assessing the client, the nurse expects which of the following findings?

Excessive urine output and decreased urine osmolality Diabetes insipidus is the result of decreased secretion of antidiuretic hormone (ADH). The client has excessive urine output, decreased urine osmolality, and serum hyperosmolarity.

A client has been diagnosed as having global aphasia. The nurse recognizes that the client will be unable to do perform which action?

Form understandable words and comprehend spoken words Global aphasia is a combination of expressive and receptive aphasia and presents a tremendous challenge to the nurse to communicate effectively with the client. In receptive and expressive aphasia, the client is unable to form words that are understandable. The client who is unable to speak at all is referred to as mute.

A client with a brain tumor experiences projectile vomiting. The nurse integrates understanding of this occurrence as resulting from which of the following?

Irritation of the medullary vagal centers

The diagnosis of multiple sclerosis is based on which test?

MRI The diagnosis of MS is based on the presence of multiple plaques in the central nervous system observed on MRI. Electrophoresis of CSF identifies the presence of oligoclonal banding. Evoked potential studies can help define the extent of the disease process and monitor changes. Neuropsychological testing may be indicated to assess cognitive impairment.

The nurse is caring for a client with a traumatic brain injury who has developed increased intracranial pressure resulting in syndrome of inappropriate antidiuretic hormone (SIADH). While assessing this client, the nurse expects which of the following findings?

Oliguria and serum hyponatremia SIADH is the result of increased secretion of antidiuretic hormone (ADH). The client becomes volume overloaded, urine output diminishes, and serum sodium concentration becomes dilute.

Which of the following, if left untreated, can lead to an ischemic stroke?

Psychosis, disorientation, delirium, insomnia, and hallucinations Advances in technology have led to the introduction of interventional neuroradiology for the treatment of aneurysms. Endovascular techniques may be used in selected clients to occlude the blood flow from the artery that feeds the aneurysm with coils or other techniques to occlude the aneurysm itself. Postoperative complications are rare but can occur. Potential complications include psychological symptoms such as disorientation, amnesia, and Korsakoff syndrome (disorder characterized by psychosis, disorientation, delirium, insomnia, hallucinations, and personality changes). Creutzfeldt-Jakob disease results in severe dementia and myoclonus. The three cardinal signs of Parkinson disease are tremor, rigidity, and bradykinesia. Huntington disease results in progressive involuntary choreiform (dancelike) movements and dementia.

Which of the following is the first-line therapy for myasthenia gravis (MG)?

Pyridostigmine bromide (Mestinon)

The nurse practitioner is able to correlate a patient's neurologic deficits with the location in the brain affected by ischemia or hemorrhage. For a patient with a left hemispheric stroke, the nurse would expect to see:

Right-sided paralysis

While providing information to a community group, the nurse tells them the primary initial symptoms of a hemorrhagic stroke are:

Severe headache and early change in level of consciousness The main presenting symptoms for ischemic stroke are numbness or weakness of the face, arm, or leg, especially on one side of the body, confusion or change in mental status, and trouble speaking or understanding speech. Severe headache, vomiting, early change in level of consciousness, and seizures are early signs of a hemorrhagic stroke. Foot drop and external hip rotation can occur if a stroke victim is not turned or positioned correctly.

Which of the following types of hematoma results from venous bleeding with blood gradually accumulating in the space below the dura?

Subdural A subdural hematoma results from venous bleeding, with blood gradually accumulating in the space below the dura. An epidural hematoma stems from arterial bleeding, usually from the middle meningeal artery, and blood accumulation above the dura. An intracerebral hematoma is bleeding within the brain that results from an open or closed head injury or from a cerebrovascular condition such as a ruptured cerebral aneurysm. A cerebral hematoma is bleeding within the skull.

ossiculoplasty

Surgical repair of an ossicle (small bone) of the middle ear.

Autonomic dysreflexia can occur with spinal cord injuries above which of the following levels?

T6 Any patient with a lesion above T6 segment is informed that autonomic dysreflexia can occur and that it may occur even years after the initial injury.

The nurse is assisting with administering a Tensilon test to a patient with ptosis. If the test is positive for myasthenia gravis, what outcome does the nurse know will occur?

Thirty seconds after administration, the facial weakness and ptosis will be relieved for approximately 5 minutes. Thirty seconds after injection, facial muscle weakness and ptosis should resolve for about 5 minutes (Hickey, 2009). Immediate improvement in muscle strength after administration of this agent represents a positive test and usually confirms the diagnosis.

A nurse is teaching a community class that those experiencing symptoms of ischemic stroke need to enter the medical system early. The primary reason for this is which of the following?

Thrombolytic therapy has a time window of only 3 hours.

A patient comes to the emergency department with severe pain in the face that was stimulated by brushing the teeth. What cranial nerve does the nurse understand can cause this type of pain?

V The trigeminal nerve (cranial nerve V) innervates the forehead, cheeks, and jaw, so pain in the face elicited when brushing the teeth would most likely involve this nerve.

aneurysm

a weakening or bulge in an arterial wall

The nurse is aware that burr holes may be used in neurosurgical procedures. Which of the following is a reason why a neurosurgeon may choose to create a burr hole in a patient?

aspiration of brain abscess Burr holes may be used in neurosurgical procedures to make a bone flap in the skull, to aspirate a brain abscess, or to evacuate a hematoma.

A client diagnosed with a stroke is ordered to receive warfarin. Later, the nurse learns that the warfarin is contraindicated and the order is canceled. The nurse knows that the best alternative medication to give is

aspirin If warfarin is contraindicated, aspirin is the best option, although other medications may be used if both are contraindicated.

trachoma

chronic, contagious form of conjunctivitis that typically leads to blindness

hypopyon

collection of inflammatory cells in the anterior chamber of the eye

persistent vegetative state

condition in which the patient is wakeful but devoid of conscious content, without cognitive or affective mental function

menieres disease

condition of the inner ear characterized by vertigo, tinnitus, fluctuating hearing loss, and pressure or pain in the affected ear

sympathetic nervous system

division of the autonomic nervous system with predominantly xcitatory responses

ptsosis

drooping eyelid

hyperopia

farsightedness

A stroke victim is experiencing memory loss and impaired learning capacity. The nurse knows that brain damage has most likely occurred in which lobe?

frontal If damage has occurred to the frontal lobe, learning capacity, memory, or other higher cortical intellectual functions may be impaired. Such dysfunction may be reflected in a limited attention span, difficulties in comprehension, forgetfulness, and a lack of motivation.

vertigo

illusion of movement

craniotomy

incision of the skull

The nurse is caring for a client who has sustained a spinal cord injury (SCI) at C5 and has developed a paralytic ileus. The nurse will prepare the client for which of the following procedures?

insertion of an ng tube Immediately after a SCI, a paralytic ileus usually develops. A nasogastric tube is often required to relieve distention and to prevent vomiting and aspiration. An enema and digital stimulation will not relieve a paralytic ileus. Bowel surgery is not necessary.

Which is a contraindication for the administration of tissue plasminogen activator (t-PA)?

intracranial hemorrhage

presbyopia

loss of accommodative power of the lens due to age

A nurse is assessing a patient's risk for pressure ulcers using the Braden scale. Which area would the nurse address?

moisture

Which nursing intervention can prevent a client from experiencing autonomic dysreflexia?

monitoring the patency of an indwelling urinary catheter A full bladder can precipitate autonomic dysreflexia, the nurse should monitor the patency of an indwelling urinary catheter to prevent its occlusion, which could result in a full bladder. Administering zolpidem tartrate, assessing laboratory values, and placing the client in Trendelenburg's position can't prevent autonomic dysreflexia.

axon

portion of the neuron that conducts impulses toward the cell body

cataract

progressive opacity of the lens of the eye

A client has undergone surgery for a spinal cord tumor that was located in the cervical area. The nurse would be especially alert for which of the following?

respiratory dysfunction When a spinal tumor is located in the cervical area, respiratory compromise may occur from postoperative edema. Hemorrhage would be a concern with any surgery. Bowel incontinence and skin breakdown are possible but not specific to cervical spinal tumors.

diplopia

seeing an object as two

tinnitus

subjective perception of sound within internal orign

infarction

tissue necrosis

A client has been diagnosed with a concussion and is to be released from the emergency department. The nurse teaches the family or friends who will be caring for the client to contact the physician or return to the ED if the client

vomits Vomiting is a sign of increasing intracranial pressure and should be reported immediately. In general, the finding of headache in a client with a concussion is an expected abnormal observation. However, a severe headache, weakness of one side of the body, and difficulty in waking the client should be reported or treated immediately.

hemiparesis

weakness of one side of the body or part of it due to an injury in the motor area of the brain

A nurse is working on a neurological unit with a nursing student who asks the difference between primary and secondary headaches. The nurse's correct response will include which of the following statements?

"A secondary headache is associated with an organic cause, such as a brain tumor." A secondary headache is a symptom associated with an organic cause, such as a brain tumor or an aneurysm. A primary headache is one for which no organic cause can be identified. These types include migraine, tension, and cluster headaches. Secondary headaches can be located in all areas of the head.

A patient undergoing rehabilitation reports problems with constipation. Which suggestion would be least appropriate?

"Keep your fluid intake to fewer than 2 liters per day."

A client with hypercholesterolemia is receiving Lipitor (atorvastatin) to prevent high cholesterol and stroke. The order is for Lipitor 40 mg PO daily. The medication is supplied in 80 mg tabs. How many tabs will the nurse administer to the client? Enter the correct number ONLY.

40 mg/80 mg = 0.5 tabs.

A college student goes to the infirmary with a fever, headache, and a stiff neck. The nurse suspects the student may have meningitis and has the student transferred to the hospital. If the diagnosis is confirmed, what should the nurse institute for those who have been in contact with this student? Select all that apply.

Administration of rifampin Administration of ceftriaxone sodium Administration of ciprofloxacin hydrochloride People in close contact with patients with meningococcal meningitis should be treated with antimicrobial chemoprophylaxis using rifampin, ciprofloxacin hydrochloride, or ceftriaxone sodium.

The nurse is performing an assessment of cranial nerve function and asks the patient to cover one nostril at a time to see if the patient can smell coffee, alcohol, and mint. The patient is unable to smell any of the odors. The nurse is aware that the patient has a dysfunction of which cranial nerve?

CN 1 Cranial nerve (CN) I is the olfactory nerve, which allows the sense of smell. Testing of CN I is done by having the patient identify familiar odors with eyes closed, testing each nostril separately. An inability to smell an odor is a significant finding, indicating dysfunction of this nerve.

The physician's office nurse is caring for a client who has a history of a cerebral aneurysm. Which diagnostic test does the nurse anticipate to monitor the status of the aneurysm?

Cerebral angiography The nurse would anticipate a cerebral angiography, which detects distortion of the cerebral arteries and veins . A myelogram detects abnormalities of the spinal canal. An electroencephalogram records electrical impulses of the brain. An echoencephalography is an ultrasound of the structures of the brain.

During assessment of a patient who has been taking dilantin for seizure management for 3 years, the nurse notices one of the side effects that should be reported. What is that side effect?

Gingival hyperplasia Side-effects of dilantin include visual problems, hirsutism, gingival hyperplasia, arrhythmias, dysarthria, and nystagmus.

A nurse is planning discharge for a client who experienced right-sided weakness caused by a stroke. During his hospitalization, the client has been receiving physical therapy, occupational therapy, and speech therapy daily. The family voices concern about rehabilitation after discharge. How should the nurse intervene?

Inform the case manager of the family's concern and provide information about the client's current clinical status so appropriate resources can be provided after discharge.

The nurse is caring for a patient with an altered LOC. What is the first priority of treatment for this patient?

Maintenance of a patent airway The first priority of treatment for the patient with altered LOC is to obtain and maintain a patent airway.

The nurse is caring for a client in the emergency department with a diagnosis of head trauma secondary to a motorcycle accident. The nurse aide is assigned to clean the client's face and torso. Which action by the nurse aide would prompt the nurse to provide further instruction?

Moving the client's head to clean behind the ears

A patient diagnosed with meningitis would be expected to exhibit which of the following clinical manifestations? Select all that apply.

Nuchal rigidity Positive Kernig's sign Positive Brudzinski's sign Photophobia Signs of meningeal irritation include nuchal rigidity (neck stiffness), a positive Kernig's sign, a positive Brudzinski's sign, and photophobia. Patients may have a fever.

What nursing intervention is appropriate for a client with receptive aphasia?

Speak slowly and clearly Nursing management of the client with receptive aphasia includes speaking slowing and clearly to assist the client in forming the sounds. Nursing management of the client with expressive aphasia includes encouraging the client to repeat sounds of the alphabet or to explore the client's ability to write. Nursing management of the client with cognitive deficits, such as memory loss, includes frequently reorienting the client to time, place, and situation.

A client is diagnosed with meningococcal meningitis. The 22-year-old client shares an apartment with one other person. What would the nurse expect as appropriate care for the client's roommate?

Treatment with antimicrobial prophylaxis as soon as possible People in close contact with clients who have meningococcal meningitis should be treated with antimicrobial chemoprophylaxis, ideally within 24 hours after exposure.

dementia

`a slowly progressive decline in mental abilities, including memory, thinking, and judgment, that is often accompanied by personality changes

migraine

a severe, unrelenting headache often accompanied by symptoms such as nausea, vomiting, and visual disturbances

reflex

an automatic response to stimuli

sympathetic opthalmia

an inflammatory condition created in the fellow eye by the affected eye

neuropathy

any disease of the nerves

When caring for a client who is post-intracranial surgery what is the most important parameter to monitor?

body temperature It is important to monitor the client's body temperature closely because hyperthermia increases brain metabolism, increasing the potential for brain damage. Therefore, elevated temperature must be relieved with an antipyretic and other measures. Extreme thirst, intake and output, and nutritional status are not the most important parameters to monitor.

contusion

bruising of the brain surface

The provider diagnoses the patient as having had an ischemic stroke. The etiology of an ischemic stroke would include which of the following?

cartiogenic emboli Aneurysms, hemorrhages, and malformations are all examples of a hemorrhagic stroke. An embolism can block blood flow, leading to ischemia.

A neurologic deficit is best defined as a deficit of the:

central and peripheral nervous systems with decreased, impaired, or absent functioning. A client with a neurologic deficit may have decreased, impaired, or absent functioning of the central and peripheral systems.

A client is admitted to an acute care facility after an episode of status epilepticus. After the client is stabilized, which factor is most beneficial in determining the potential cause of the episode?

compliance with prescribed medication regimen The most common cause of status epilepticus is sudden withdraw of anticonvulsant therapy. The type of medication prescribed, the client's stress level, and weight change don't contribute to this condition.

injection

congestion of blood vessels

When the nurse observes that the client has extension and external rotation of the arms and wrists and plantar flexion of the feet, the nurse records the client's posture as

decerebrate Decerebrate posturing is the result of lesions at the midbrain and is more ominous than decorticate posturing. The client's head and neck arch backward, and the muscles are rigid. In decorticate posturing, which results from damage to the nerve pathway between the brain and spinal cord and is also very serious, the client has flexion and internal rotation of the arms and wrists, as well as extension, internal rotation, and plantar flexion of the feet.

astigmatism

defective curvature of the cornea or lens of the eye

dysarthria

defects of articulation due to neurologic causes

spondylosis

degenerative changes occurring in a disc and adjacent vertebral bodies; can occur in the cervical or lumbar vertebrae

A nurse is monitoring a client for increasing intracranial pressure (ICP). Early signs of increased ICP include:

diminished responsiveness Usually, diminished responsiveness is the first sign of increasing ICP. Pupillary changes occur later. Increased ICP causes systolic blood pressure to rise. Temperature changes vary and may not occur even with a severe decrease in responsiveness.

A patient has been diagnosed with myasthenia gravis. The nurse documents the initial and most common manifestation of:

diplopia The initial manifestation of MG usually involves the ocular muscles. Diplopia (double vision) and ptosis (drooping of the eyelids) are common. The majority of patients also experience weakness of the muscles of the face and throat, generalized weakness, and weakness of the facial muscles.

otorrhea

drainage from ear

rhinirrhea

drainage from the nose

A client with Parkinson's disease has been receiving levodopa as treatment for the past 7 years. The client comes to the facility for an evaluation and the nurse observes facial grimacing, head bobbing, and smacking movements. The nurse interprets these findings as which of the following?

dyskinesia Most clients within 5 to 10 years of taking levodopa develop a response to the medication called dyskinesia, manifested as facial grimacing, rhythmic jerking movements of the hands, head bobbing, chewing and smacking movements, and involuntary movements of the trunk and extremities. Bradykinesia refers to an overall slowing of active movement and is a manifestation of the disorder. Micrographia refers to the development of small handwriting as dexterity declines with Parkinson's disease. Dysphonia refers to soft, slurred, low-pitched, and less audible speech that occurs as the disorder progresses.

The nursing instructor is teaching about hematomas to a pre-nursing pathophysiology class. What would the nursing instructor describe as an arterial bleed with rapid neurologic deterioration?

epidural hematoma An epidural hematoma stems from arterial bleeding, usually from the middle meningeal artery, and blood accumulation above the dura. It is characterized by rapidly progressive neurologic deterioration.

status epilepticus

episode in which the patient experiences multiple seizure bursts with no recovery time in between

The rehabilitation nurse is caring for a 25-year-old client who suffered extensive injuries in a motorcycle accident. During each interaction with the client, what action should the nurse perform most frequently?

evaluate positioning

After a motor vehicle crash, a client is admitted to the medical-surgical unit with a cervical collar in place. The cervical spinal X-rays haven't been read, so the nurse doesn't know whether the client has a cervical spinal injury. Until such an injury is ruled out, the nurse should restrict this client to which position?

flat, except for logrolling as needed When caring for the client with a possible cervical spinal injury who's wearing a cervical collar, the nurse must keep the client flat to decrease mobilization and prevent further injury to the spinal column. The client can be logrolled, if necessary, with the cervical collar on.

aqueous humor

fluid in the eye, found between the cornea and the lens

middle ear effusion

fluid in the middle ear without evidence of infection

anterior chamber

fluid-filled space between the cornea and iris

A patient sustained a head injury during a fall and has changes in personality and affect. What part of the brain does the nurse recognize has been affected in this injury?

frontal The frontal lobe, the largest lobe, located in the front of the brain. The major functions of this lobe are concentration, abstract thought, information storage or memory, and motor function. It contains Broca's area, which is located in the left hemisphere and is critical for motor control of speech. The frontal lobe is also responsible in large part for a person's affect, judgment, personality, and inhibitions (Hickey, 2009).

Which lobe of the brain is responsible for concentration and abstract thought?

frontal The major functions of the frontal lobe are concentration, abstract thought, information storage or memory, and motor function. The parietal lobe analyzes sensory information such as pressure, vibration, pain, and temperature. The occipital lobe is the primary visual cortex. The temporal lobe contains the auditory receptive areas located around the temples.

what is the most common type of meningiomas?

grade I meningioma

glaucoma

group of conditions characterized by increased intraocular pressure

A 37-year-old mother of three has just been diagnosed with a grade I meningioma. As part of patient education, the nurse tells the patient that:

growth is slow and symptoms are caused by compression rather than tissue invasion.

neovascularizaton

growth of abnormal new blood vessels

The nurse is assisting a patient to sit up on the side of the bed in preparation for standing. The patient has been on strict bedrest for more than a week. While assuming the sitting position, the patient begins to report feeling dizzy and nauseated. The patient is pale and diaphoretic. Which of the following would the nurse do next?

have the patient lie back down The patient is exhibiting signs of orthostatic hypotension and cerebral insufficiency from the change in position. The best action would be have the patient lie back down because he or she is not tolerating the change in position. Taking deep breaths would be ineffective in raising the patient's blood pressure or increasing the blood supply to the brain. Having the patient stand up immediately would worsen the patient's symptoms. Using a transfer board would have no effect on the patient's symptoms, which are from the change in position.

The nurse is caring for a 35-year-old man whose severe workplace injuries necessitate bilateral below-the-knee amputations. How should the nurse anticipate that the client will respond to this news?

he client will experience grief in an individualized manner. Loss of limb is a profoundly emotional experience, which the client will experience in a subjective manner, and largely unpredictable, manner. Psychotherapy may or may not be necessary. It is not possible to accurately predict the sequence or timing of the client's grief. The client may or may not benefit from psychotherapy.

The nurse is assessing a client with meningitis. Which of the following signs would the nurse expect to observe?

headache & nuchal rigitity Headache and fever are the initial symptoms of meningitis. Nuchal rigidity can be an early sign. Photophobia is also a well-recognized sign in meningitis. Ptosis and diplopia are usually seen with myasthenia gravis. Hyporeflexia in the legs is seen with Guillain-Barre syndrome.

secondary headache

headache identified as a symptom of another organic disorder (e.g., brain tumor, hypertension)

A critical care nurse is caring for a client with a cerebrovascular accident (CVA) The client is prescribed heparin for treatment. The nurse knows to monitor the client for what adverse effects?

hemorrhage A client with a CVA who is given heparin should be monitored for hemorrhage and bleeding at the subcutaneous injection site. Respiratory distress, high blood pressure, or migraine attacks are not likely to occur in such a client.

dyskinesia

impaired ability to execute voluntary movements

expressive aphasia

inability to express oneself (damage to left frontal lobe)

aphasia

inability to express oneself or to understand language

A client with a spinal cord injury and subsequent urine retention receives intermittent catheterization every 4 hours. The average catheterized urine volume has been 550 ml. The nurse should plan to:

increase the frequency of the catheterizations As a rule of practice, if intermittent catheterization for urine retention typically yields 500 ml or more, the frequency of catheterization should be increased. Indwelling catheterization is less preferred because of the risk of urinary tract infection and the loss of bladder tone. Fluid restrictions aren't indicated in this case; the problem isn't overhydration, rather it's urine retention. A condom catheter doesn't help empty the bladder of the client with urine retention.

The nurse is caring for a client with a ventriculostomy. Which assessment finding demonstrates effectiveness of the ventriculostomy?

increased icp is 12 mmHg

Babinski reflex

indicative of abnormalities in the motor control pathways

labyrinthitis

inflammation of the labyrinth (inner ear)

While assessing the client at the beginning of the shift, the nurse inspects a surgical dressing covering the operative site after the clients' cervical discectomy. The nurse notes that the drainage is 75% saturated with serosanguineous discharge. What is the nurse's most appropriate action?

inform the surgeon of the possibility of a dural leak After a cervical discectomy, the nurse will monitor the operative site and dressing covering this site. Serosanguineous drainage may indicate a dural leak. This constitutes a risk for meningitis, but is not a direct sign of infection. This should be reported to the surgeon, not just reinforced and observed.

primary injury

initial damage to the brain that results from the traumatic event

endophthalmitis

intraocular infection

nystagmmus

involuntary oscillation of the eyeball

brain death

irreversible loss of brain function

A patient comes to the emergency department with a large scalp laceration after being struck in the head with a glass bottle. After assessment of the patient, what does the nurse do before the physician sutures the wound?

irrigates the wound to remove debris Scalp wounds are potential portals of entry for organisms that cause intracranial infections. Therefore, the area is irrigated before the laceration is sutured to remove foreign material and to reduce the risk for infection.

A client with epilepsy is having a seizure. What intervention should the nurse do after the seizure?

keep the client on one side The nurse will need to keep the client on one side to prevent aspiration. Make sure the airway is patent. On awakening, reorient the client to the environment. If the client is confused or wandering, guide the client gently to a bed or chair. If the client becomes agitated after a seizure (postictal), stay a distance away, but close enough to prevent injury until the client is fully aware. The client does not need a cooling blanket after a seizure. The client's temperature should not be elevated from the seizure. The nurse should not pry the client's mouth open after a seizure so that the airway remains open.

ataxia

lack of muscle coordination

flaccidity

lack of muscle tone

A client is exhibiting signs of increasing intracranial pressure (ICP). Which intravenous solution (IV) would the nurse anticipate hanging?

lactated ringer's With increasing ICP, isotonic normal saline, lactated Ringer's, or hypertonic (3%) saline solutions are used to decrease swelling in the brain cells. D5W, 0.45% NSS, and 0.33% NSS are all hypotonic solutions that will move more fluid into the cells, worsening the ICP.

deafness

loss of the ability to hear

Bone density testing will be completed for the client with post-polio syndrome. The nurse teaches the client bone density testing is used to identify what potential complication?

low bone mass and osteoporosis Bone density testing in clients with post-polio syndrome has demonstrated low bone mass and osteoporosis. Thus, the importance of identifying risks, preventing falls, and treating osteoporosis must be discussed with clients and their families.

If a client has a lower motor neuron lesion, the nurse would expect to observe which manifestation upon physical assessment?

low muscle tone A client with a lower motor neuron lesion would be expected to have decreased muscle tone. Those with upper motor neuron lesion would have hyperactive reflexes, no muscle atrophy, and muscle spasticity.

A nurse is caring for a client with a cerebral aneurysm. Which nursing interventions would be most useful to the nurse to avoid bleeding in the brain? Select all that apply.

maintain the head f the bed at 30 report changes in neuro status as soon as a worsening trend is identified avoid any activities that cause a valsalva maneuver Cerebral aneurysm precautions are implemented for the patient with a diagnosis of aneurysm to provide a nonstimulating environment, prevent increases in intracranial pressure, and prevent further bleeding. The patient is placed on bed rest in a quiet, nonstressful environment, because activity, pain, and anxiety are thought to elevate the blood pressure, which may increase the risk for bleeding. The head of the bed is elevated 30 degrees to promote venous drainage and decrease intracranial pressure. Any activity that suddenly increases the blood pressure or obstructs venous return is avoided. This includes the Valsalva maneuver, straining, forceful sneezing, pushing oneself up in bed and acute flexion or rotation of the head and neck (which compromises the jugular veins). Stool softeners and mild laxatives are prescribed to prevent constipation, which can cause an increase in intracranial pressure. Dim lighting is helpful for photophobia. Increasing fluid volume does not affect brain bleeding.

myopia

nearsightedness

Which clinical manifestation would be exhibited by a client following a hemorrhagic stroke of the right hemisphere?

neglect of the left side

emmetropia

normal vision

tbi open/penetrating

occurs when an object penetrates the skull, enters the brain, and damages the soft brain tissue in its path (penetrating injury), or when blunt trauma to the head is so severe that it opens the scalp, skull, and dura to expose the brain

photophobia

pain on exposure to light

seizures

paroxysmal transient disturbance of the brain resulting from a discharge of abnormal electrical activity

chorea

rapid, jerky, involuntary, purposeless movements of the extremities or facial muscles, including facial grimacing

A nurse is completing discharge teaching for the client who has left-sided hemiparesis following a stroke. When investigating the client's home environment, the nurse should focus on which nursing diagnosis?

risk for injury

A client admitted with a cerebral contusion is confused, disoriented, and restless. Which nursing diagnosis takes the highest priority?

risk for injury related to neuro deficit Because a cerebral contusion causes altered cognition, the nurse should identify Risk for injury related to neurologic deficit as the primary nursing diagnosis and focus on interventions that promote client safety and prevent further injury. Disturbed sensory perception (visual) related to neurologic trauma, Feeding self-care deficit related to neurologic trauma, and Impaired verbal communication related to confusion are pertinent but don't take precedence over client safety.

The nurse is caring for a patient having a hemorrhagic stroke. What position in the bed will the nurse maintain this patient?

semi-fowlers The head of the bed is elevated 15 to 30 degrees (semi-Fowler's position) to promote venous drainage and decrease intracranial pressure.

otalgia

sensation of fullness or pain in the ear

The initial symptoms of variant Creutzfeldt-Jakob disease (vCJD) include

sensory disturbance, limb pain, and behavioral changes

A nurse completes the Glasgow Coma Scale on a patient with traumatic brain injury (TBI). Her assessment results in a score of 6, which is interpreted as:

severe TBI A score of 13 to 15 is classified as mild TBI, 9 to 12 is moderate TBI, and 3 to 8 is severe TBI. A score of 3 indicates severe impairment of neurologic function, deep coma, brain death, or pharmacologic inhibition of the neurologic response; a score of 8 or less typically indicates an unconscious patient; a score of 15 indicates a fully alert and oriented patient.

Which are characteristics of autonomic dysreflexia?

severe hypertension, slow heart rate, pounding headache, sweating Autonomic dysreflexia is an exaggerated sympathetic nervous system response. Hypertension, tachycardia, bradycardia, and flushed skin would occur.

exostoses

small, hard, bony protrusions in the lower posterior bony portion of the ear canal

A family member brings the client to the clinic for a follow-up visit after a stroke. The family member asks the nurse what he can do to decrease his chance of having another stroke. What would be the nurse's best answer?

stop smoking as soon as possible Smoking is a modifiable and highly significant risk factor for stroke. The significance of smoking, and the potential benefits of quitting, exceed the roles of sodium, diet, and regular medical assessments.

The nurse is caring for a client with traumatic brain injury (TBI). Which clinical finding, observed during the reassessment of the client, causes the nurse the most concern?

temperature increase from 98 to 99.6 Fever in the client with a TBI can be the result of damage to the hypothalamus, cerebral irritation from hemorrhage, or infection. The nurse monitors the client's temperature every 2 to 4 hours. If the temperature increases, efforts are made to identify the cause and to control it using acetaminophen and cooling blankets to maintain normothermia. The other clinical findings are within normal limits.

Romberg test

test for cerebellar dysfunction that cam be done with the patient seated or standing (20 seconds is positive)

Cushing's response

the brain's attempt to restore blood flow by increasing arterial pressure to overcome the increased intracranial pressure

A client is actively hallucinating during an assessment. The nurse would be correct in documenting the hallucination as a disturbance in

thought content Hallucinations are disturbances of thought content. They are not disturbances in motor ability, intellectual function, or emotional status.

A patient who has experienced a stroke is learning to use a cane to ambulate. The patient has left-sided weakness. After teaching the patient about using the cane, the nurse determines that the patient has understood the instructions when stating that using the cane on the right is done for which purpose?

to distribute weight away from the affected side Holding a cane on the uninvolved side distributes weight away from the involved side. Holding the cane close to the body prevents leaning. Using a cane won't promote a long stride length or reduce the risk of edema.

The nurse is assessing the client's pupils following a sports injury. Which of the following assessment findings indicates a neurologic concern? Select all that apply.

unequal pupils absence of pupillary response pinpoint of pupils Normal assessment findings includes that the pupils are equal and reactive to light. Pupils that are unequal, pinpoint in nature, or fail to respond indicate a neurologic impairment.

A client is hospitalized with Guillain-Barré syndrome. Which nursing assessment finding is most significant?

uneven labored respirations

akinetic mutism

unresponsiveness to the environment, makes no movement or sound but sometimes opens eyes

The community health nurse is conducting a home visit with a client who was discharged from hospital 3 days ago after surgical resection of a brain tumor and radiation therapy. The client is accompanied by his partner during the nurse's visit. During the visit, the client's partner becomes tearful. How should the nurse respond?

"Going through this experience with your partner has been very difficult for you, I'm sure. Can you tell me about your experience so far?" Nurses caring for clients with brain tumors need to be able to understand the difficulties the caregivers experience. The study by Coolbrandt et al. (2015) found that multiple caregivers felt unprepared to care for the client but stated no one ever questioned the demands on them. To fully understand what the caregiver may need to continue to support the client through treatment toward end of life, the nurse should aim to capture the individual perspective of the care provider. In this case, the nurse only asks closed-ended questions in the alternate answer options. In these answer options, the nurse also makes assumptions about why the client's partner is tearful. In the correct answer, the nurse asks an open-ended question, demonstrating genuine curiosity about the caregiver's experience. This type of communication alone can help the caregiver feel less isolated and more supported.

A nurse is discussing a lumbar puncture with a nursing student who observed the procedure. The student noticed that the cerebrospinal fluid was blood tinged and asks what that means. The correct reply is which of the following?

"It can mean a traumatic puncture or a subarachnoid bleed." The needle is inserted below the level of the spinal cord, which prevents damage to the cord. The cerebral spinal fluid (CSF) should be clear and colorless. Pink or bloody CSF may indicate a subarachnoid bleed or local trauma from the puncture. The hypothalamus is located deep inside the brain and does not affect the color of the CSF.

A client with newly diagnosed seizures asks about stigma associated with epilepsy. The nurse will respond with which of the following statements?

"Many people with developmental disabilities resulting from neurologic damage also have epilepsy." Many people who have developmental disabilities because of serious neurologic damage also have epilepsy. Epilepsy is not associated with intellectual level. It is not synonymous with intellectual disability or mental illness.

A client with a spinal cord injury is to receive Lovenox (enoxaparin) 50 mg subcutaneously twice a day. The medication is supplied in vials containing 80 mg per 0.8 mL. How many mL will constitute the correct dose? Enter the correct number ONLY.

0.5 (50 mg/80 mg) X 0.8 mL = 0.5 mL.

A nurse is reviewing a patient's laboratory test results. Which serum albumin level would lead the nurse to suspect that the patient is at risk for pressure ulcers?

2.5 g/mL Serum albumin is a sensitive indicator of protein deficiency. Levels below 3 g/mL are associated with hypoalbuminemic tissue edema and increased risk of pressure ulcers.

A patient is admitted to the hospital with an ICP reading of 20 mm Hg and a mean arterial pressure of 90 mm Hg. What would the nurse calculate the CPP to be?

70 mmHg Changes in ICP are closely linked with cerebral perfusion pressure (CPP). The CPP is calculated by subtracting the ICP from the mean arterial pressure (MAP). For example, if the MAP is 100 mm Hg and the ICP is 15 mm Hg, then the CPP is 85 mm Hg. The normal CPP is 70 to 100 mm Hg

A client with cerebral metastasis suddenly experiences a seizure for which phenytoin 10 mg/kg intravenously is ordered as an initial loading dose. The client weighs 165 pounds. How many milligrams of phenytoin should the client receive? Enter the number ONLY.

750 First, change the client's weight in pounds to kilograms by dividing the weight by 2.2 (2.2 pounds = 1 kg). The client's weight is 75 kg. Next, set up a proportion: 10/1 = x/75; cross multiply and solve for x, which is 750.

A client is receiving intravenous (IV) mannitol to prevent increased intracranial pressure. The order is for mannitol 1.5 grams per kg of body weight IV now. The client weighs 143 lbs (65 kg). How many grams will the nurse administer to the client? Enter the correct number in tenths.

97.5 143 lbs/2.2 = 65 kg. 65 kg x 1.5 = 97.5 grams.

A nurse is assisting with a community screening for people at high risk for stroke. To which of the following clients would the nurse pay most attention?

A 60-year-old Black man The 60-year-old Black man has three risk factors: gender, age, and race. Black people have almost twice the incidence of first stroke compared with White people.

The nurse is taking care of a client with a history of headaches. The nurse takes measures to reduce headaches and administer medications. Which appropriate nursing interventions may be provided by the nurse to such a client?

Apply warm or cool cloths to the forehead or back of the neck Applying warm or cool cloths to the forehead or back of the neck and massaging the back relaxes muscles and provides warmth to promote vasodilation. These measures are aimed at reducing the occurrence of headaches in the client. A client with transient ischemic attacks is advised to maintain hydration and drink eight glasses of fluid a day. A Heimlich maneuver is performed to clear the airway if the client cannot speak or breathe after swallowing food. The nurse uses pressure-relieving pads or a similar type of mattress to maintain peripheral circulation in the client's body.

Which nutritional deficiency may delay wound healing?

C Vitamins A, C, and K; pyridoxine; riboflavin; and thiamin are necessary for wound healing. Adequate protein intake is necessary for improving skin integrity. Vitamin D and calcium are necessary for bone healing. Vitamin E isn't necessary for wound healing.

Which condition is a rare, transmissible, progressive fatal disease of the central nervous system characterized by spongiform degeneration of the gray matter of the brain?

Creutzfeldt-Jakob disease Creutzfeldt-Jakob disease causes severe dementia and myoclonus. Multiple sclerosis is a chronic, degenerative, progressive disease of the central nervous system characterized by the occurrence of small patches of demyelination in the brain and spinal cord. Parkinson disease is associated with decreased levels of dopamine due to destruction of pigmented neuronal cells in the substantia nigra in the basal ganglia of the brain. Huntington disease is a chronic, progressive, hereditary disease of the nervous system that results in progressive involuntary dancelike movements and dementia.

After a stroke, a client is admitted to the facility. The client has left-sided weakness and an absent gag reflex. He's incontinent and has a tarry stool. His blood pressure is 90/50 mm Hg, and his hemoglobin is 10 g. Which nursing intervention is a priority for this client?

Elevating the head of the bed to 30 degrees Because the client's gag reflex is absent, elevating the head of the bed to 30 degrees helps minimize the client's risk of aspiration. Checking the stools, performing ROM exercises, and keeping the skin clean and dry are important, but preventing aspiration through positioning is the priority.

Which safety action will the nurse implement for a client receiving oxygen therapy who is undergoing magnetic resonance imaging (MRI)?

Ensure that no client care equipment containing metal enters the room where the MRI table is located. For client safety the nurse must make sure that no client care equipment that contains metal or metal parts (e.g., portable oxygen tanks) enters the room where the MRI is located. The client must be assessed for the presence of medication patches with foil backing (e.g., nicotine patch) that may cause a burn. The magnetic field generated by the unit is so strong that any metal-containing items will be strongly attracted and can literally be pulled away with such great force that they can fly like projectiles toward the magnet.

A client undergoes cerebral angiography for evaluation of a subarachnoid hemorrhage. Which findings indicate spasm or occlusion of a cerebral vessel by a clot?

Hemiplegia, seizures, and decreased level of consciousness Spasm or occlusion of a cerebral vessel by a clot causes signs and symptoms similar to those of a stroke: hemiplegia, seizures, decreased level of consciousness, aphasia, hemiparesis, and increased focal symptoms. Nausea, vomiting, and profuse sweating suggest a delayed reaction to the contrast medium used in cerebral angiography. Difficulty breathing or swallowing may signal a hematoma in the neck. Tachycardia, tachypnea, and hypotension suggest internal hemorrhage

A nurse helps a patient recently diagnosed with a pituitary adenoma understand that:

Most tumors produce too much of one or more hormones. The majority of these tumors are benign. In rare cases, they may be malignant. Functioning tumors produce hormones, frequently in excessive amounts, resulting in conditions such as hyperthyroidism, Cushing's syndrome, and gigantism or acromegaly.

Which is the initial diagnostic test for a stroke?

Noncontrast computed tomography The initial diagnostic test for a stroke is nonconstrast computed tomography performed emergently to determine whether the event is ischemic or hemorrhagic. Further diagnostics include a carotid Doppler, electrocardiogram, and transcranial Doppler.

The nurse who is employed in a neurologist's office is performing a history and assessment on a client experiencing hearing difficulty. The nurse is most correct to gather equipment to assess the function of which cranial nerve?

VIII There are 12 pairs of cranial nerves. Cranial nerve VIII is the vestibulocochlear or auditory nerve responsible for hearing and balance. Cranial nerve II is the optic nerve. Cranial nerve VI is the abducens nerve responsible for eye movement. Cranial nerve XI is the accessory nerve and is involved with head and shoulder movement.

otosclerosis

a condition characterized by abnormal spongy bone formation around the stapes

The nurse is caring for a client hospitalized with a severe exacerbation of myasthenia gravis. When administering medications to this client, what is a priority nursing action?

administer meds at the exact intervals as ordered The nurse must administer medications at the exact intervals ordered to maintain therapeutic blood levels and prevent symptoms from returning. Assessing the client's reaction, documenting medication and dose, and giving the client plenty of fluids are not the priority nursing action for this client.

Which of the following diagnostic studies provides visualization of cerebral blood vessels?

cerebral angiography Cerebral angiography provides visualization of cerebral blood vessels and can localize most cerebral trauma. A PET scan measures the brain's activity and is useful in differentiating tumor from scar tissue or radiation necrosis. Cytologic studies of the cerebral spinal fluid (CSF) may be performed to detect malignant cells because central nervous system tumors can shed cells into the CSF. Computer-assisted stereotactic biopsy is being used to diagnose deep-seated brain tumors.

Locked-in syndrome

condition resulting from a lesion in the pons in which the patient lacks all distal motor activity (paralysis) but cognition is intact

keratoconus

cone-shaped deformity of the cornea

Low levels of the neurotransmitter serotonin lead to which of the following disease processes?

depression A decrease of serotonin leads to depression. A decrease in the amount of acetylcholine causes myasthenia gravis. Parkinson's disease is caused by a depletion of dopamine. Decreased levels of GABA may cause seizures.

The trochlear nerve controls which function?

eye muscle movement The trochlear nerve coordinates the muscles that move the eye. The acoustic nerve functions in hearing and equilibrium. The optic nerve functions in visual acuity and visual fields. The hypoglossal nerve functions in the movement of the tongue.

A client's spouse relates how the client reported a severe headache and then was unable to talk or move their right arm and leg. After diagnostics are completed and the client is admitted to the hospital, when would basic rehabilitation begin?

immediately Beginning basic rehabilitation during the acute phase is an important nursing function. Measures such as position changes and prevention of skin breakdown and contractures are essential aspects of care during the early phase of rehabilitation. The nursing goal is to prevent complications that may interfere with the client's potential to recover function.

An emergency department nurse is interviewing a client who is presenting with signs of an ischemic stroke that began 2 hours ago. The client reports a history of a cholecystectomy 6 weeks ago and is taking digoxin, warfarin, and labetalol. What factor poses a threat to the client for thrombolytic therapy?

international normal ratio greater than 2 The client is at risk for further bleeding if the international normalized ratio is greater than 2. Thrombolytic therapy must be initiated within 3 hours in clients with ischemic stroke. The client is not eligible for thrombolytic therapy if she has had surgery within 14 days. Digoxin and labetalol do not prohibit thrombolytic therapy.

nystagmus

involuntary rhythmc eye movement

What is the only known risk factor for brain tumors?

ionizing radiation

agnosia

loss of ability to recognize familiar objects

sensorineural hearing loss

loss of hearing related to damage to the end organ for hearing or cranial nerve VIII or both

sudden unexpected death in epilepsy

nontraumatic nondrowning unexpected death of a patient with epilepsy

binocular vision

normal ability of both eyes to focus on one object and fuse two images into one

The most common cause of cholinergic crisis includes which of the following

overmedication A cholinergic crisis, which is essentially a problem of overmedication, results in severe generalized muscle weakness, respiratory impairment, and excessive pulmonary secretion that may result in respiratory failure. Myasthenic crisis is a sudden, temporary exacerbation of MG symptoms. A common precipitating event for myasthenic crisis is infection. It can result from undermedication.

sciatica

pain that follows the pathway of the sciatic nerve, caused by compression or trauma of the nerve or its roots

hemiplegia

paralysis of one side of the body

dendrite

portion of the neuron that conducts impulses toward the cell body

presbycusis

progressive hearing loss associated with aging

parasympathetic nervous system

the division of the autonomic nervous system that calms the body, conserving its energy

tetraplegia

varying degrees of paralysis of both arms and legs, with dysfunction of bowel and bladder from a lesion of the cervical segments of the spinal cord; formerly called quadriplegia

A 30-year-old was diagnosed with amyotrophic lateral sclerosis (ALS). Which statement by the client would indicate a need for more teaching from the nurse?

"My children are at greater risk to develop this disease." There is no known cause for ALS, and no reason to suspect genetic inheritance. ALS usually begins with muscle weakness of the arms and progresses. The client is encouraged to remain active for as long as possible to prevent respiratory complications.

A client recently experienced a stroke with accompanying left-sided paralysis. His family voices concerns about how to best interact with him. They report the client doesn't seem aware of their presence when they approach him on his left side. What advice should the nurse give the family?

"The client is unaware of his left side. You should approach him on the right side." The client is experiencing unilateral neglect and is unaware of his left side. The nurse should advise the family to approach him on his unaffected (right) side. Approaching the client on the affected side would be counterproductive. It's too premature to make the determination whether this condition will be permanent.

The parents of a client intubated due to the progression of Guillain-Barré syndrome ask whether their child will die. What is the best response by the nurse?

"There are no guarantees, but a large portion of people with Guillain-Barré syndrome survive." The survival rate of Guillain-Barré syndrome is approximately 90%. The client may make a full recovery or suffer from some residual deficits. Telling the parents not to worry dismisses their feelings and does not address their concerns. Progression of Guillain-Barré syndrome to the diaphragm does not significantly decrease the survival rate, but it does increase the chance of residual deficits. The family should be given information about Guillain-Barré syndrome and the generally favorable prognosis. With no prognosis offered, the parents are not having their concerns addressed.

The nurse is assessing the client's mental status . Which question will the nurse include in the assessment?

"Who is the president of the United States?" Assessing orientation to time, place, and person assists in evaluating mental status. Does the client know what day it is, what year it is, and the name of the president of the United States? Is the client aware of where he or she is? Is the client aware of who the examiner is and why he or she is in the room? "Can you write your name on this piece of paper?" will assess language ability. "Can you count backward from 100?" assesses the client's intellectual function. "Are you having hallucinations?" assesses the client's thought content.

The nurse is caring for a patient who was involved in a motor vehicle accident and sustained a head injury. When assessing deep tendon reflexes (DTR), the nurse observes diminished or hypoactive reflexes. How will the nurse document this finding?

1+ Diminished or hypoactive DTRs are indicated by a score of 1+, no response by a score of 0, a normal response by a score of 2+, and an increased response by a score of 3+.

Vagus nerve demyelinization, which may occur in Guillain-Barré syndrome, would not be manifested by which of the following?

20/20 vision

blindness is defined as

20/400 or less

A client has been hospitalized for diagnostic testing. The client has just been diagnosed with multiple sclerosis, which the physician explains is an autoimmune disorder. How would the nurse explain an autoimmune disease to the client?

A disorder in which killer T cells and autoantibodies attack or destroy natural cells—those cells that are "self" Autoimmune disorders are those in which killer T cells and autoantibodies attack or destroy natural cells—those cells that are "self." Autoantibodies, antibodies against self-antigens, are immunoglobulins. They target histocompatible cells, cells whose antigens match the person's own genetic code. Autoimmune disorders are not caused by too many or too few immunoglobulins, and histocompatible cells do not attack immunoglobulins in an autoimmune disorder.

Which of the following is the most common side effect of tissue plasminogen activator (tPA)?

Bleeding Bleeding is the most common side effect of tPA. The patient is closely monitored for bleeding (at IV insertion sites, gums, urine/stools, and intracranially by assessing changes in level of consciousness). Headache, increased ICP, and hypertension are not side effects of tPA.

A 76-year-old client is brought to the clinic by his daughter. The daughter states that her father has had two transient ischemic attacks (TIAs) in the past week. The physician orders carotid angiography, and the report reveals that the carotid artery has been narrowed by atherosclerotic plaques. What treatment option does the nurse expect the physician to offer this client to increase blood flow to the brain?

Carotid endarterectomy If narrowing of the carotid artery by atherosclerotic plaques is the cause of the TIAs, a carotid endarterectomy (surgical removal of atherosclerotic plaque) is a treatment option. A balloon angioplasty, a procedure similar to a percutaneous transluminal coronary artery angioplasty, is performed to dilate the carotid artery and increase blood flow to the brain. Options A, B, and C are not surgical options to increase blood flow through the carotid artery to the brain.

The nurse is caring for a client with an inoperable brain tumor. What teaching is important for the nurse to do with these clients?

Explaining hospice care and services The nurse explains hospice care and services to clients with brain tumors that no longer are at a stage where they can be cured. Managing muscle weakness and offering family support groups are important but explaining hospice is the best answer. Optimizing nutrition at this point is not a priority.

A client is admitted reporting low back pain. How will the nurse best determine if the pain is related to a herniated lumbar disc?

Have the client lie on the back and lift the leg, keeping it straight. A client who can lie on the back and raise a leg in a straight position will have pain radiating into the leg if there is a herniated lumbar disc. This action stretches the sciatic nerve. The client may also have muscle weakness and decreased tendon reflexes and sensory loss. The client should still be able to walk, and have bowel movements, so this assessment will not assist the nurse to confirm the diagnosis. Pain on ambulation is also not specific to this condition.

The nurse is performing an initial assessment on a client admitted with a possible brain abscess. Which of the following would the nurse most likely find?

Headache that is worse in the morning The most prevailing symptom of a brain abscess is headache, which is usually worse in the early morning. Ptosis and diplopia are seen in clients with myasthenia gravis. Nuchal rigidity is seen in clients with meningitis.

Which term will the nurse use when referring to blindness in the right or left half of the visual field in both eyes?

Homonymous hemianopsia

Which terms refers to blindness in the right or left half of the visual field in both eyes?

Homonymous hemianopsia

The nurse has been educating a client newly diagnosed with MS. Which statement by the client indicates an understanding of the education?

I will stretch daily as directed by the physical therapist A stretching routine should be established. Stretching can help prevent contractures and muscle spasticity. Hot baths are discouraged because of the risk of injury. Clients have sensory loss that may contribute to the risk of burns. In addition, hot temperatures may cause an increase in symptoms. Warm packs should be encouraged to provide relief. Progressive weight-bearing exercises are effective in managing muscle spasms. Clients should not hurry through the exercise activity because it may increase muscle spasticity.

What is the function of cerebrospinal fluid (CSF)?

It cushions the brain and spinal cord. CSF is produced primarily in the lateral ventricles of the brain. It acts as a shock absorber and cushions the spinal cord and brain against injury caused by sudden or extreme movement. CSF also functions in the removal of waste products from cerebral tissue. CSF doesn't act as an insulator or a barrier and it doesn't produce cerebral neurotransmitters.

A client in the intensive care unit (ICU) has a traumatic brain injury. The nurse must implement interventions to help control intracranial pressure (ICP). Which of the following are appropriate interventions to help control ICP?

Keep the client's neck in a neutral position (no flexing). To assist in controlling ICP in clients with severe brain injury, the following are recommended: elevate the head of the bed as prescribed (gravity helps drain fluid), maintain head/neck in neutral alignment (no twisting or flexing), give sedation as ordered to prevent agitation, and avoid noxious stimuli (scatter procedures so that client does not become overtired).

A client has experienced an ischemic stroke that has damaged the lower motor neurons of the brain. Which of the following deficits would the nurse expect during assessment?

Lack of deep tendon reflexes Damage to the occipital lobe can result in visual agnosia, whereas damage to the temporal lobe can cause auditory agnosia. If damage has occurred to the frontal lobe, learning capacity, memory, or other higher cortical intellectual functions may be impaired. Such dysfunction may be reflected in a limited attention span, difficulties in comprehension, forgetfulness, and lack of motivation. Damage to the lower motor neurons may cause decreased muscle tone, flaccid muscle paralysis, and a decrease in or loss of reflexes.

A client is admitted with weakness, expressive aphasia, and right hemianopia. The brain MRI reveals an infarct. The nurse understands these symptoms to be suggestive of which of the following findings?

Left CVA When the infarct is on the left side of the brain, the symptoms are likely to be on the right, and the speech is more likely to be involved. If the MRI reveals an infarct, TIA is no longer the diagnosis. There is not enough information to determine if the stroke is still evolving or is complete.

A client with weakness and tingling in both legs is admitted to the medical-surgical unit with a tentative diagnosis of Guillain-Barré syndrome. On admission, which assessment is most important for this client?

Lung auscultation and measurement of vital capacity and tidal volume In Guillain-Barré syndrome, polyneuritis commonly causes weakness and paralysis, which may ascend to the trunk and involve the respiratory muscles. Lung auscultation and measurement of vital capacity, tidal volume, and negative inspiratory force are crucial in detecting and preventing respiratory failure — the most serious complication of polyneuritis. A peripheral nerve disorder, polyneuritis doesn't cause increased ICP. Although the nurse must evaluate the client for pain and discomfort and must assess the nutritional status and metabolic state, these aren't priorities.

A patient 3 days postoperative from a craniotomy informs the nurse, "I feel something trickling down the back of my throat and I taste something salty." What priority intervention does the nurse initiate?

Notify the physician of a possible cerebrospinal fluid leak. Any sudden discharge of fluid from a cranial incision is reported at once, because a large leak requires surgical repair. Attention should be paid to the patient who complains of a salty taste or "postnasal drip," because this can be caused by cerebrospinal fluid trickling down the throat.

The nurse is caring for a patient with Huntington's disease in the long-term care facility. What does the nurse recognize as the most prominent symptom of the disease that the patient exhibits?

Rapid, jerky, involuntary movements The most prominent clinical features of the disease are chorea (rapid, jerky, involuntary, purposeless movements), impaired voluntary movement, intellectual decline, and often personality changes

A client on your unit is scheduled to have intracranial surgery in the morning. Which nursing intervention helps to avoid intraoperative complications, reduce cerebral edema, and prevent postoperative vomiting?

Restrict fluids before surgery. Before surgery, the nurse should restrict fluids to avoid intraoperative complications, reduce cerebral edema, and prevent postoperative vomiting. The nurse administers prescribed medications such as an anticonvulsant phenytoin, like Dilantin, to reduce the risk of seizures before and after surgery, an osmotic diuretic, and corticosteroids. Preoperative sedation is omitted.

A client is hospitalized when presenting to the emergency department with right-sided weakness. Within 6 hours of being admitted, the neurologic deficits had resolved and the client was back to his presymptomatic state. The nurse caring for the client knows that the probable cause of the neurologic deficit was what?

TIA

Which interventions would be recommended for a client with dysphagia? Select all that apply.

Test the gag reflex before offering food or fluids Allow ample time to eat. Assist the client with meals Interventions for dysphagia include placing food on the unaffected side of the mouth, allowing ample time to eat, assisting the client with meals, and testing the client's gag reflex before offering food or fluids.

The nurse assists the health care provider (HCP) in completing a lumbar puncture (LP). Which should the nurse note as a concern?

The cerebrospinal fluid (CSF) is cloudy in nature. The CSF is normally clear and colorless; therefore, CSF that is cloudy would be noted by the nurse as a concern. The HCP is correct to maintain aseptic procedure. At 90 mm H20, the client's CSF fluid pressure falls within normal limits (between 80 and 100 mm H20). Sometimes the HCP will administer medication via intrathecal injection during an LP, which should not be a cause for concern.

The nurse is caring for a postoperative client who had surgery to decrease intracranial pressure after suffering a head injury. Which assessment finding is promptly reported to the physician?

The client's vital signs are temperature, 100.9° F; heart rate, 88 beats/minute; respiratory rate, 18 breaths/minute; and blood pressure, 138/80 mm Hg. The assessment finding promptly reported to the physician is the information which may cause complications. It is important to report the elevation in client temperature (100.9° F) because hyperthermia increases brain metabolism, increasing the potential for brain damage. It is not unusual for the client to experience periorbital edema and ecchymosis secondary to the head injury and surgery. Improved level of consciousness is a positive outcome of the treatment provided. There is no complication related to semi-Fowler's position.

A patient with myasthenia gravis is in the hospital for treatment of pneumonia. The patient informs the nurse that it is very important to take pyridostigmine bromide on time. The nurse gets busy and does not administer the medication until after breakfast. What outcome will the patient have related to this late dose?

The muscles will become fatigued and the patient will not be able to chew food or swallow pills. Maintenance of stable blood levels of anticholinesterase medications, such as pyridostigmine, is imperative to stabilize muscle strength. Therefore, the anticholinesterase medications must be administered on time. Any delay in administration of medications may exacerbate muscle weakness and make it impossible for the patient to take medications orally.

The nurse is providing information about strokes to a community group. Which of the following would the nurse identify as the primary initial symptoms of an ischemic stroke?

Weakness on one side of the body and difficulty with speech The main presenting symptoms for an ischemic stroke are numbness or weakness of the face, arm, or leg, especially on one side of the body; confusion or change in mental status; and trouble speaking or understanding speech. Severe headache, vomiting, early change in level of consciousness, and seizures are early signs of a hemorrhagic stroke. Foot drop and external hip rotation are things that can occur if a stroke victim is not turned or positioned correctly.

The nurse educator is providing orientation to a new group of staff nurses on an oncology unit. Part of the orientation is to help nurses understand the differences between various types of brain tumors. The nurse educator correctly identifies that glioma tumors are classified based on the fact that they originate where in the brain?

Within the brain tissue The most aggressive type of malignant brain tumor is a glioma, which originates within the brain tissue.

incomplate spinal cord lesion

a condition in which there is preservation of the sensory or motor fibers, or both, below the lesion

complete spinal cord lesion

a condition that involves total loss of sensation and voluntary muscle control below the lesion

What diet can the nurse recommend to a patient with hypoproteinemia that spares protein?

a diet high in carbs

autonomic dysreflexia

a life-threatening emergency in spinal cord injury patients that causes a hypertensive emergency; also called autonomic hyperreflexia

The nurse is seeing a client who has just been diagnosed with a meningioma. The client states he is confused because the provider stated, "If you have to be diagnosed with a brain tumor, this is the least harmful." The client asks the nurse for clarification. How should the nurse respond?

a motor cortex tumor

prion

a pathogen smaller than a virus that is resistant to standard sterilization procedures

minimally conscious state

a state in which the patient demonstrates awareness but cannot communicate thoughts or feelings

concussion

a temporary loss of neurologic function with no apparent structural damage to the brain

clonus

abnormal movement marked by alternating contraction and relaxation of a muscle occurring in rapid succession

delirium

acute confused state

position/postural sense

awareness of position of parts of the body without looking at them

neurogenic bladder

bladder dysfunction that results in urinary retention or bladder overactivity

scotomas

blind or partially blind areas in the visual field

hemianopsia

blindness in half the visual field

hyphema

blood in the anterior chamber

When caring for a client who is post-intracranial surgery, what is the most important parameter to monitor?

body temperature It is important to monitor the client's body temperature closely; hyperthermia increases brain metabolism, increasing the potential for brain damage. Therefore, elevated temperature must be relieved with an antipyretic and other measures. Options A, B, and C are not the most important parameters to monitor.

A client experienced a stroke that damaged the hypothalamus. The nurse should anticipate that the client will have problems with:

body temperature control The body's thermostat is located in the hypothalamus; therefore, injury to that area can cause problems with body temperature control. Balance and equilibrium problems are related to cerebellar damage. Visual acuity problems would occur following occipital or optic nerve injury. Thinking and reasoning problems are the result of injury to the cerebrum.

A client with a spinal cord injury has full head and neck control when the injury is at which level?

c5 At level C5, the client retains full head and neck control. At C1 the client has little or no sensation or control of the head and neck. At C2 to C3 the client feels head and neck sensation and has some neck control. At C4 the client has good head and neck sensation and motor control

A client has sustained a traumatic brain injury with involvement of the hypothalamus. The nurse is concerned about the development of diabetes insipidus. Which of the following would be an appropriate nursing intervention to monitor for early signs of diabetes insipidus?

daily weights A record of daily weights is maintained for the client with a traumatic brain injury, especially if the client has hypothalamic involvement and is at risk for the development of diabetes insipidus. A weight loss will alert the nurse to possible fluid imbalance early in the process.

coma

deep prolonged unconsciousness

neurodegenerative

deterioration of cells or function of the nervous system

A client arrives at the emergency department complaining of extreme muscle weakness after minimal effort. The physician suspects myasthenia gravis. Which drug will be used to test for this disease?

edrophonium Edrophonium temporarily blocks the breakdown of acetylcholine, thus increasing acetylcholine level in the blood, and relieves weakness. Because of its short duration of action, edrophonium is the drug of choice for diagnosing myasthenia gravis. It's also used to differentiate myasthenia gravis from cholinergic toxicity. Ambenonium is used as an antimyasthenic. Pyridostigmine serves primarily as an adjunct in treating severe anticholinergic toxicity; it's also an antiglaucoma agent and a miotic. Carbachol reduces intraocular pressure during ophthalmologic procedures; topical carbachol is used to treat open-angle and closed-angle glaucoma.

A client with a brain tumor is complaining of a headache upon awakening. Which nursing action would the nurse take first?

elevate the head of the bed The first action would be to elevate the head of the bed to promote venous drainage of blood and cerebral spinal fluid (CSF). Then, a neurological assessment would be completed to determine if any other assessment findings are significant of increasing intracranial pressure (ICP). The administering of routine ordered drugs is not a priority, and narcotic analgesics would be avoided in clients with ICP issues.

pseudobulbar affect

emotional disturbance characterized by uncontrollable episodes of crying or laughing, or other emotional displays

Level of consciousness (LOC) can be assessed based on criteria in the Glasgow Coma Scale (GCS). Which of the following indicators are assessed in the GCS? Select all that apply.

eye opening motor response verbal response LOC can be assessed based on the criteria in the GCS, which include eye opening, verbal response, and motor response. The patient's responses are rated on a scale from 3 to 15. Intelligence and muscle strength are not measured in the GCS.

Which is often the most disabling clinical manifestation of multiple sclerosis?

fatigue Fatigue affects 87% of people with MS, and 40% of that group indicate that fatigue is the most disabling symptom. Pain, spasticity, and ataxia are other clinical manifestations of MS, but are not the most disabling.

Lower motor neuron lesions cause

flaccid muscles Lower motor neuron lesions cause flaccidity, muscle atrophy, decreased muscle tone, and loss of voluntary control. Upper motor neuron lesions cause increased muscle tone. Upper motor neuron lesions cause no muscle atrophy. Upper motor neuron lesions cause hyperactive and abnormal reflexes.

Which is a nonmodifiable risk factor for ischemic stroke?

gender Nonmodifiable risk factors include gender, age, and race. Modifiable risk factors include atrial fibrillation, hyperlipidemia, and smoking.

A patient arrives to have an MRI done in the outpatient department. What information provided by the patient warrants further assessment to prevent complications related to the MRI?

i am trying to quit smoking and have a patch on Before the patient enters the room where the MRI is to be performed, all metal objects and credit cards (the magnetic field can erase them) must be removed. This includes medication patches that have a metal backing and metallic lead wires; these can cause burns if not removed (Bremner, 2005).

A patient with a fractured left fibula is being taught how to use crutches. Which statement by the patient indicates that the teaching was effective?

i need to allow my arms and hands to support my body weight

A diagnostic test has determined that the appropriate diet for the client with a left cerebrovascular accident (CVA) should include thickened liquids. Which of the following is the priority nursing diagnosis for this client?

impaired swallowing Impaired Swallowing was evident on the video fluoroscopy. Aspiration, Malnutrition, and Decreased Fluid Volume Risk can occur but are not the primary diagnosis at this point in time.

acute otitis media

inflammation in the middle ear lasting less than 6 weeks

A client with a concussion is discharged after the assessment. Which instruction should the nurse give the client's family?

look for signs of increased intracranial pressure The nurse informs the family to monitor the client closely for signs of increased intracranial pressure if findings are normal and the client does not require hospitalization. Signs of increased intracranial pressure include headache, blurred vision, vomiting, and lack of energy or sleepiness. The nurse looks for a halo sign to detect any cerebrospinal fluid drainage.

Pressure ulcers may begin within hours of an acute spinal cord injury (SCI) and may cause delay of rehabilitation, adding to the cost of hospitalization. The most effective approach is prevention. Which of the following nursing interventions will most protect the client against pressure ulcers?

meticulous cleanliness Meticulous cleanliness is the best choice for preventing pressure ulcers. A continuous indwelling catheter is not conducive to preventing pressure ulcers. Pressure-sensitive areas should be kept well lubricated with lotion. The client does not know the best positioning techniques for prevention of skin breakdown. The nurse and client together should decide how to best position the body.

paraplegia

paralysis of the lower extremities with dysfunction of the bowel and bladder from a lesion in the thoracic, lumbar, or sacral region of the spinal cord

hyperemia

red eyes resulting from dilation of the vasculature of the conjunctiva

The nurse teaches the client that corticosteroids will be used to treat his brain tumor to

reduce cerebral edema Corticosteroids may be used before and after treatment to reduce cerebral edema and to promote a smoother, more rapid recovery. Corticosteroids do not prevent extension of the tumor or facilitate regeneration of neurons. Stereotactic procedures identify the precise location of the tumor.

The nurse has completed evaluating the client's cranial nerves. The nurse documents impairment of the right cervical nerves (CN IX and CN X). Based on these findings, the nurse should instruct the client to

refrain from eating & drinking for now Significant findings of CN IX (glossopharyngeal) include difficulty swallowing (dysphagia) and impaired taste, and significant findings of CN X (vagus) include weak or absent gag reflex, difficulty swallowing, aspiration, hoarseness, and slurred speech (dysarthria). Based on these findings, the nurse should instruct the client to refrain from eating and drinking and should contact the health care provider. The other instructions are associated with abnormalities of CN II (optic) and CN VIII (acoustic).

evisceration

removal of the contents of the eye while leaving the sclera and cornea intact

enucleation

removal of the eyeball and part of the optic nerve

chronic otitis media

repeated episodes of acute otitis media causing irreversible tissue damage and persistent tympanic membrane perforation

A nurse is developing a plan of care for an 85-year-old woman who is bedridden following a stroke. Which of the following would the nurse be least likely to include in the plan of care for this patient to reduce her risk for pressure ulcers?

repositioning the patient once a shift

A client admitted to the emergency department is being evaluated for the possibility of a stroke. Which assessment finding would lead the nurse to suspect that the client is experiencing a hemorrhagic stroke?

severe exploding headache

Which condition occurs when blood collects between the dura mater and arachnoid membrane?

subdural hematoma A subdural hematoma is a collection of blood between the dura mater and brain, space normally occupied by a thin cushion of fluid. Intracerebral hemorrhage is bleeding in the brain or the cerebral tissue with the displacement of surrounding structures. An epidural hematoma is bleeding between the inner skull and the dura, compressing the brain underneath. An extradural hematoma is another name for an epidural hematoma.

A patient who has a disability is attempting to gain employment via vocational rehabilitation. What should the nurse closely monitor in the patient with a disability attempting to seek employment?

substance abuse Substance abuse is a critical issue in rehabilitation, especially for people with disabilities who are attempting to gain employment via vocational rehabilitation. The rates of substance abuse, including alcohol abuse, in people with disabilities are two to four times as high as in the general population, and this increased abuse is associated with numerous risks that may have an adverse impact.

transsphenoidal

surgical approach to the pituitary via the sphenoid sinuses

exenteration

surgical removal of the entire contents of the orbit

tympanoplasty

surgical repair of the tympanic membrane

spasticity

sustained increase in tension of a muscle when it is passively lengthened or stretched

papilledema

swelling of the optic disc

Which cerebral lobe contains the auditory receptive areas?

temporal The temporal lobe plays the most dominant role of any area of the cortex in cerebration. The frontal lobe, the largest lobe, controls concentration, abstract thought, information storage or memory, and motor function. The parietal lobe contains the primary sensory cortex, which analyzes sensory information and relays interpretation to the thalamus and other cortical areas. The occipital lobe is responsible for visual interpretation.

A patient with Huntington's disease is prescribed medication to reduce the chorea. What medication will the nurse administer that is the only drug approved for the treatment of this symptom?

tetrabenazine (xenazine)

A nurse is conducting a presentation about brain cancer for a local community group. During the presentation, one of the group members asks, "What causes brain tumors?" Which response by the nurse would be most appropriate?

the cause of most brain tumors is still not really known

Which client should the nurse assess for degenerative neurologic symptoms?

the client with huntington disease Huntington disease is a chronic, progressive, degenerative neurologic hereditary disease of the nervous system that results in progressive involuntary choreiform movement and dementia. Paget disease is a musculoskeletal disorder, characterized by localized rapid bone turnover, most commonly affecting the skull, femur, tibia, pelvic bones, and vertebrae. Osteomyelitis is an infection of the bone. Malignant glioma is the most common type of brain tumor.

A client with increased intracranial pressure has a cerebral perfusion pressure (CPP) of 40 mm Hg. How should the nurse interpret the CPP value?

the cpp is low The normal CPP is 70 to 100 mm Hg. Therefore, a CPP of 40 mm Hg is low. Changes in intracranial pressure (ICP) are closely linked with cerebral perfusion pressure (CPP). The CPP is calculated by subtracting the ICP from the mean arterial pressure (MAP). Patients with a CPP of less than 50 mm Hg experience irreversible neurologic damage.

When should the nurse plan the rehabilitation of a patient who is having an ischemic stroke?

the day the patient has a stroke Although rehabilitation begins on the day the patient has the stroke, the process is intensified during convalescence and requires a coordinated team effort.

autonomic nervous system

the part of the peripheral nervous system that controls the glands and the muscles of the internal organs

craniectomy

the surgical removal of a portion of the skull

vitreous humor

the transparent jellylike tissue filling the eyeball behind the lens.

monro-kellie hypothesis

theory that states that due to limited space for expansion within the skull, an increase in any one of the cranial contents—brain tissue, blood, or cerebrospinal fluid—causes a change in the volume of the others; also referred to as Monro-Kellie doctrine

The nurse is admitting a client from the emergency department with a reported spinal cord injury. What device would the nurse expect to be used to provide correct vertebral alignment and to increase the space between the vertebrae in a client with spinal cord injury?

traction with weights and pulleys Traction with weights and pulleys is applied to provide correct vertebral alignment and to increase the space between the vertebrae. A cast and a cervical collar are used to immobilize the injured portion of the spine. A turning frame is used to change the client's position without altering the alignment of the spine.

trichiasis

turning in of the eyelashes

A client diagnosed with Huntington disease is on a disease-modifying drug regimen and has a urinary catheter in place. Which potential complication is the highest priority for the nurse while monitoring the client?

urinary tract infection Because all disease-modifying drug regimens for Huntington disease can decrease immune cells and infection protection, it is most important for the nurse to assess for acquired infections such as urinary tract infections, especially if the client is catheterized. Severe depression is common and can lead to suicide. Symptoms of Huntington disease develop slowly and include mental apathy and emotional disturbances, choreiform movements (uncontrollable writhing and twisting of the body), grimacing, difficulty chewing and swallowing, speech difficulty, intellectual decline, and loss of bowel and bladder control. Assessing for these other conditions is appropriate but not as important as assessing for urinary tract infection in the client on a disease-modifying drug regimen with a urinary catheter in place.

The nurse is fitting a patient for crutches that are required for an ankle injury. What quick method can the nurse use to measure so that the crutches will be of appropriate height?

use the patients height and subtract 16 inches

altered level of consciousness

when a patient is not oriented, does not follow commands, or needs persistent stimuli to achieve a state of alertness

Cerebral edema peaks at which time point after intracranial surgery?

24 hours Cerebral edema tends to peak 24 to 36 hours after surgery.

A patient has been diagnosed with meningococcal meningitis at a community living home. When should prophylactic therapy begin for those who have had close contact with the patient?

24 hrs People in close contact with patients with meningococcal meningitis should be treated with antimicrobial chemoprophylaxis using rifampin (Rifadin), ciprofloxacin hydrochloride (Cipro), or ceftriaxone sodium (Rocephin). Therapy should be started within 24 hours after exposure because a delay in the initiation of therapy limits the effectiveness of the prophylaxis.

The nurse is caring for a client who is to have a lumbar puncture. What are the lowest vertebrae that contain the spinal cord?

2nd lumbar vertebrae The spinal cord ends between the first and second lumbar vertebrae.

A nurse assesses the patient's LOC using the Glasgow Coma Scale. What score indicates severe impairment of neurologic function?

3 LOC, a sensitive indicator of neurologic function, is assessed based on the criteria in the Glasgow Coma Scale: eye opening, verbal response, and motor response (Barlow, 2012). The patient's responses are rated on a scale from 3 to 15. A score of 3 indicates severe impairment of neurologic function, brain death, or pharmacologic inhibition of the neurologic response. A score of 15 indicates that the patient is fully responsive

An emergency department nurse is awaiting the arrival of a client with signs of an ischemic stroke that began 1 hour ago, as reported by emergency medical personnel. The treatment window for thrombolytic therapy is which of the following?

3 hours Rapid diagnosis of stroke and initiation of thrombolytic therapy (within 3 hours) in clients with ischemic stroke leads to a decrease in the size of the stroke and an overall improvement in functional outcome after 3 months.

A client is receiving intravenous (IV) dobutamine (Dobutrex) to help provide adequate perfusion to the brain. The order is for dobutamine 50 mg in 500 mL D5W at 2 mcg/kg/min. The client weighs 58 kg. At how many mL per hour will the nurse administer this medication? Enter the correct number ONLY.

70 58 kg X 2 = 116 mcg/min. 116 mcg X 60 minutes = 6,960 mcg per hour. 6,960 mcg/1000 = 6.96 mg, rounded to 7 mg/hour. (7 mg/50 mg) X 500 mL = 70 mL/hour.

The nurse is caring for a client postoperatively from a spinal tumor resection. The nurse assesses that the client has partial paralysis. What anticipated problems should the nurse include in the client's care plan? Select all that apply.

BP = 175/45 mm Hg; HR = 42 bpm With a blood pressure of 175/45 mm Hg, it is evident that this client is experiencing progressively rising ICP, resulting from an advanced stage of the brain tumor. This blood pressure demonstrates a wide pulse pressure, meaning the difference between systolic and diastolic pressure is large. A heart rate of 42 bpm indicates the client is bradycardic. This finding paired with hypertensive blood pressure with a widening pulse pressure are part of the Cushing triad related to increased ICP.

A client is following up after a visit to the emergency department where testing indicated that the client had suffered a transient ischemic attack. What lifestyle changes would the nurse include in teaching to prevent further reoccurrence? Select all that apply.

Blood pressure control Smoking cessation Weight loss Primary prevention of ischemic stroke remains the best approach. A healthy lifestyle including not smoking, engaging in physical activity (at least 40 minutes a day, 3 to 4 days a week), maintaining a healthy weight, and following a healthy diet (including modest alcohol consumption) can reduce the risk of having a stroke. Specific diets that have decreased risk of stroke include the Dietary Approaches to Stop Hypertension (DASH) diet (high in fruits and vegetables, moderate in low-fat dairy products, and low in animal protein), the Mediterranean diet (supplemented with nuts), and overall diets that are rich in fruits and vegetables. Research findings suggest that low-dose aspirin may lower the risk of a first stroke for those who are at risk.

The nurse is caring for a client following a spinal cord injury who has a halo device in place. The client is preparing for discharge. Which statement by the client indicates the need for further instruction?

I can apply powder un the liner to help with swelling Powder is not used inside the vest because it may contribute to the development of pressure ulcers. The areas around the four pin sites of a halo device are cleaned daily and observed for redness, drainage, and pain. The pins are observed for loosening, which may contribute to infection. If one of the pins becomes detached, the head is stabilized in a neutral position by one person while another notifies the neurosurgeon. The skin under the halo vest is inspected for excessive perspiration, redness, and skin blistering, especially on the bony prominences. The vest is opened at the sides to allow the torso to be washed. The liner of the vest should not become wet because dampness can cause skin excoriation. The liner should be changed periodically to promote hygiene and good skin care.

A comatose client is being cared for by a critical care nurse who documents that the client responds only to very painful stimuli by fragmentary, delayed reflex withdrawal. The nurse knows that reflexes in the body are centered where?

In the spinal cord The spinal cord functions as a passageway for ascending sensory and descending motor neurons. Its two main functions are to provide centers for reflex action and to serve as a pathway for impulses to and from the brain. Reflex centers are not in the pons, the medulla, or the midbrain.

Which of the following is accurate regarding a hemorrhagic stroke?

Main presenting symptom is an "exploding headache." One of hemorrhagic stroke's main presenting symptom is an "exploding headache." In ischemic stroke, functional recovery usually plateaus at 6 months; it may be caused by a large artery thrombosis and may have a presenting symptoms of numbness or weakness of the face.

herniation

abnormal protrusion of an organ or other body structure through a defect or natural opening

The nurse in the emergency department is caring for a patient brought in by the rescue squad after falling from a second-story window. The nurse assesses ecchymosis over the mastoid and clear fluid from the ears. What type of skull fracture is this indicative of?

basilar skull fracture A fracture of the base of the skull is referred to as a basilar skull fracture. Fractures of the base of the skull tend to traverse the paranasal sinus of the frontal bone or the middle ear located in the temporal bone. Therefore, they frequently produce hemorrhage from the nose, pharynx, or ears, and blood may appear under the conjunctiva. An area of ecchymosis (bruising) may be seen over the mastoid (Battle's sign). Basilar skull fractures are suspected when CSF escapes from the ears (CSF otorrhea) and the nose (CSF rhinorrhea).

Which is the earliest sign of increasing intracranial pressure?

change in level of consciousness The earliest sign of increasing intracranial pressure (ICP) is a change in level of consciousness. Other manifestations of increasing ICP are vomiting, headache, and posturing.

A nurse is caring for a client with a diagnosis of trigeminal neuralgia. Which activity is altered as a result of this diagnosis?

chewing Trigeminal neuralgia is a painful condition that involves the fifth (V) cranial nerve (the trigeminal nerve) and is important to chewing.

refraction

determination of the refractive errors of the eye for the purpose of vision correction

dysphagia

difficulty swallowing

endolymphatic hydrops

dilation of the endolymphatic space of the inner ear

The nurse is assessing a client newly diagnosed with myasthenia gravis. Which of the following signs would the nurse most likely observe?

diplopia and ptsosis The initial manifestation of myasthenia gravis involves the ocular muscles, such as diplopia and ptosis. The remaining choices relate to multiple sclerosis.

Impaired balance and uncontrolled tremors of Parkinson's disease is correlated with which neurotransmitter?

dopamine The impaired balance and uncontrolled tremors of Parkinson's disease have been linked with low levels of dopamine. The other neurotransmitters have not been implicated in Parkinson's disease in this manner.

conductive hearing loss

loss of hearing in which efficient sound transmission to the inner ear is interrupted by some obstruction or disease process

The nurse is educating a client with myasthenia gravis about medications. The nurse is sure to include which of the following?

meds must be taken on time If medications are not taken on time, exacerbations may occur, making it impossible for the client to take the medication orally. Medications must always be taken with the client upright to avoid aspiration. Procaine (Novocain) should be avoided and the client's dentist must be informed.

Which cranial nerve is responsible for muscles that move the eye and lids?

oculomotor The oculomotor (III) cranial nerve is also responsible for pupillary constriction and lens accommodation. The trigeminal (V) cranial nerve is responsible for facial sensation, corneal reflex, and mastication. The vestibulocochlear (VIII) cranial nerve is responsible for hearing and equilibrium. The facial (VII) nerve is responsible for salivation, tearing, taste, and sensation in the ear.

The nurse is assisting a patient in assuming a side-lying position. What intervention would be best for the nurse to provide?

place the uppermost hip slightly forward in a position of slight abduction

intracranial pressure

pressure exerted by the volume of the intracranial contents within the cranial vault

transection

severing of the spinal cord, transection can be complete or incomplete

cholesteatoma

tumor of the middle ear or mastoid, or both, that can destroy structures of the temporal bone

ectropion

turning out of the lower eyelid

A nurse is describing the concept of rehabilitation to a group of families who have members in need of these services. Which statement would the nurse include in the description?

"Rehabilitation focuses on the person's abilities." Rehabilitation focuses on abilities, not disabilities. It begins with the initial contact with the patient. The goal is to restore the patient's ability to function independently or at a pre-illness or pre-injury level of functioning as quickly as possible. If this is not possible, the aims are to maximize independence and prevent secondary disability as well as to promote a quality of life acceptable to the patient. It includes the use of adaptive and assistive devices to promote the greatest level of independence possible.

The nurse is providing care for a client who has limited mobility after a stroke. In order to assess the client for contractures, the nurse should assess the client's:

ROM Each joint of the body has a normal range of motion. To assess a client for contractures, the nurse should assess whether the client can complete the full range of motion. Assessing DTRs, muscle size, or joint pain does not reveal the presence or absence of contractures.

A client has been transferred to a rehabilitative setting from an acute care unit. What is the most important reason for the nurse to begin a program for activities of daily living (ADLs) as soon as the client is admitted to a rehabilitation facility?

ability to perform ADL's may be the key to reentering the community

exophthalmos

abnormal protrusion of the eyeball

Which term refers to the failure to recognize familiar objects perceived by the senses?

agnosia

dizziness

altered sensation of orientation in space

chemosis

edema of the conjunctiva

Which of the following areas of the brain are responsible for temperature regulation?

hypothalamus The hypothalamus also controls and regulates the autonomic nervous system and maintains temperature by promoting vasoconstriction or vasodilation. The thalamus acts primarily as a relay station for all sensation except smell. The medulla and pons are essential for respiratory function.

A patient was body surfing in the ocean and sustained a cervical spinal cord fracture. A halo traction device was applied. How does the patient benefit from the application of the halo device?

it allows for stabilization of the cervical spine along with early ambulation Halo devices provide immobilization of the cervical spine while allowing early ambulation

tbi closed/blunt

occurs when the head accelerates and then rapidly decelerates or collides with another object and brain tissue is damaged, but there is no opening through the skull and dura

When educating a patient about the use of antiseizure medication, what should the nurse inform the patient is a result of long-term use of the medication in women?

osteoporosis Because of bone loss associated with the long-term use of antiseizure medications, patients receiving antiseizure agents should be assessed for low bone mass and osteoporosis. They should be instructed about strategies to reduce their risks of osteoporosis

A nurse is providing education about migraine headaches to a community group. The cause of migraines has not been clearly demonstrated, but is related to vascular disturbances. A member of the group asks about familial tendencies. The nurse's correct reply will be which of the following?

there is a strong familial tendency Migraine headaches have a strong familial tendency. Migraines are primary headaches, not secondary headaches.

The nurse is preparing the client for a diagnostic test to evaluate blood flow within intracranial blood vessels. For which test is the nurse preparing the client?

transcranial doppler Transcranial Doppler flow studies are used to study a tumor's blood flow within intracranial blood vessels. Cerebral angiography may be used to study a tumor's blood supply or obtain information about vascular lesions. Magnetic resonance imaging provides information similar to that provided by computed tomography, but with improved tissue contrast, resolution, and anatomic definition, and it examines the lesion in multiple planes.

decerebration

an abnormal body posture associated with a severe brain injury, characterized by extreme extension of the upper and lower extremities

decortication

an abnormal posture associated with severe brain injury, characterized by abnormal flexion of the upper extremities and extension of the lower extremities

A nurse caring for a patient with head trauma will be monitoring the patient for Cushing's triad. What will the nurse recognize as the symptoms associated with Cushing's triad? Select all that apply.

bradypnea, hypertension, bradycardia At a certain point as intracranial pressure increases due to an injury, the brain's ability to autoregulate becomes ineffective and decompensation (ischemia and infarction) begins. When this occurs, the patient exhibits significant changes in mental status and vital signs. The bradycardia, hypertension, and bradypnea associated with this deterioration are known as Cushing's triad, which is a grave sign.

A patient who has suffered a stroke is unable to maintain respiration and is intubated and placed on mechanical ventilator support. What portion of the brain is most likely responsible for the inability to breathe?

brain stem The brain stem consists of the midbrain, pons, and medulla oblongata (see Fig. 65-2). Portions of the pons help regulate respiration. Motor fibers from the brain to the spinal cord and sensory fibers from the spinal cord to the brain are located in the medulla. Reflex centers for respiration, blood pressure, heart rate, coughing, vomiting, swallowing, and sneezing are located in the medulla.

A client with fungal encephalitis receiving IV amphotericin B reports fever, chills, and body aches. What action by the nurse is appropriate?

diphenhydramine & acetaminophen Administration of amphotericin B may cause fever, chills, and body aches. The administration of diphenhydramine and acetaminophen approximately 30 minutes before the administration of amphotericin B may prevent these side effects. Monitoring the serum creatinine and blood urea nitrogen levels may alert the nurse to the development of renal insufficiency and the need to address the client's renal status, but the symptoms listed are not consistent with renal alterations. Because the symptoms listed are associated with the administration of amphotericin B, obtaining CSF fluid for culture is not indicated at this time.

A 52-year-old married man with two adolescent children is beginning rehabilitation following a motor vehicle accident. The nurse planning the client's care. Who will the client's condition affect?

him and his entire family Clients and families who suddenly experience a physically disabling event or the onset of a chronic illness are the ones who face several psychosocial adjustments, even if the client recovers completely.

external otitis

inflammation of the external ear

A patient was admitted to a rehabilitation unit for treatment of a spinal cord injury. The admitting diagnosis is central cord syndrome. During an admissions physical, the nurse expects to find:

loss of motor power and sensation in the upper extremities Characteristics of a central cord injury include motor deficits (in the upper extremities compared to the lower extremities; sensory loss varies but is more pronounced in the upper extremities); bowel/bladder dysfunction is variable, or function may be completely preserved.

A nurse and nursing student are caring for a client recovering from a lumbar puncture yesterday. The client reports a headache despite being on bedrest overnight. The physician plans an epidural blood patch this morning. The student asks how this will help the headache. The correct reply from the nurse is which of the following?

"The blood will seal the hole in the dura and prevent further loss of cerebral spinal fluid." Loss of CSF causes the headache. Occasionally, if the headache persists, the epidural blood patch technique may be used. Blood is withdrawn from the antecubital vein and injected into the site of the previous puncture. The rationale is that the blood will act as a plug to seal the hole in the dura and prevent further loss of CSF. The blood is not put into the subarachnoid space. The needle is inserted below the level of the spinal cord, which prevents damage to the cord. It is not a lack of moisture that prevents healing; it is more related to the size of the needle used for the puncture.

When planning care for a client with a head injury, which position should the nurse include in the care plan to enhance client outcomes?

30 degree head elevation For clients with increased intracranial pressure (ICP), the head of the bed should be elevated to 30 degrees to promote venous outflow. Trendelenburg's position is contraindicated because it can raise ICP. Flat or neutral positioning is indicated when elevating the head of the bed would increase the risk of neck injury or airway obstruction. A side-lying position isn't specifically a therapeutic treatment for increased ICP.

A patient is admitted via ambulance to the emergency room of a stroke center at 1:30 p.m. with symptoms that the patient said began at 1:00 p.m. Within 1 hour, an ischemic stroke had been confirmed and the doctor ordered tPA. The nurse knows to give this drug no later than what time?

4:00 pm Tissue plasminogen activator (tPA) must be given within 3 hours after symptom onset. Therefore, since symptom onset was 1:00 pm, the window of opportunity ends at 4:00 pm.

A client is admitted with a cervical spine injury sustained during a diving accident. When planning this client's care, the nurse should assign highest priority to which nursing diagnosis?

Ineffective breathing pattern Because a cervical spine injury can cause respiratory distress, the nurse should take immediate action to maintain a patent airway and provide adequate oxygenation. Impaired physical mobility, Disturbed sensory perception (tactile), and Dressing or grooming self-care deficit may be appropriate for a client with a spinal cord injury — particularly during the course of recovery — but they don't take precedence over a diagnosis of Ineffective breathing pattern.

Which is a chronic, degenerative, progressive disease of the central nervous system characterized by the occurrence of demyelination in the brain and spinal cord?

MS The cause of MS is not known, and the disease affects twice as many women as men. Parkinson disease is associated with decreased levels of dopamine caused by destruction of pigmented neuronal cells in the substantia nigra in the basal ganglia of the brain. Huntington disease is a chronic, progressive, hereditary disease of the nervous system that results in progressive involuntary dancelike movements and dementia. Creutzfeldt-Jakob disease is a rare, transmissible, progressive fatal disease of the CNS characterized by spongiform degeneration of the gray matter of the brain.

A rehabilitation nurse is preparing a presentation for clients and caregivers about issues that clients with disabilities may face. Which of the following would be most appropriate for the nurse to include in the presentation?

Priority setting is helpful in dealing with the impact of the disability. For clients with disabilities, the nurse would emphasize the use of coping strategies and teach the patient how to cope with the disability through priority setting. A loss of sexual functioning does not necessarily correspond to a loss of sexual feeling. Rather, the physical and emotional aspects of sexuality, despite the physical loss of function, continue to be important for people with disabilities. The fatigue associated with disabilities results from numerous factors, such as the physical and emotional demands, as well as the ineffective coping, unresolved grief, disordered sleep patterns, and depression. Although the most obvious care tasks after discharge involve physical care, other elements of the caregiving role include psychosocial support and a commitment to this supportive role.

A nurse has taught a client how to perform quadriceps-setting exercises. The nurse determines that the client has understood the instructions when he demonstrates which of the following?

Pushes the popliteal area against the mattress while raising the heel The client demonstrates quadriceps-setting exercises by attempting to push the popliteal area against the mattress and at the same time raising the heel. With gluteal setting exercises, the client contracts the buttocks together for a count of five and then relaxes them for a count of five. With push-up exercises, the client raises the body by pushing the hands against the chair seat or mattress while he is in a sitting position. For pull-up exercises, the client lifts the body off the mattress while holding onto a trapeze while in bed or raises the arms above the head then lowers them while holding weights.

The nurse is performing a neurological assessment of a client who has sustained damage to the frontal cortex. Which of the following deficits will the nurse look for during assessment?

The inability to tell how a mouse and a cat are alike The client with damage to the frontal cortex will display a deficit in intellectual functioning. Questions designed to assess this capacity might include the ability to recognize similarities: for example, how are a mouse and dog or pen and pencil alike? The Romberg test assesses balance, which has to do with the cerebellar and basal ganglia influence on the motor system. Absence of movement below the waist suggests a deficit with the spinal cord. Intentional tremors have to do with deficits of the motor system.

A client was hit in the head with a ball and knocked unconscious. Upon arrival at the emergency department and subsequent diagnostic tests, it was determined that the client suffered a subdural hematoma. The client is becoming increasingly symptomatic. How would the nurse expect this subdural hematoma to be classified?

acute Subdural hematomas are classified as acute, subacute, and chronic according to the rate of neurologic changes. Symptoms progressively worsen in a client with an acute subdural hematoma within the first 24 hours of the head injury.

A client is transferred to the intensive care unit after evacuation of a subdural hematoma. Which nursing intervention reduces the client's risk of increased intracranial pressure (ICP)?

administering a stool softener as ordered To prevent the client from straining at stool, which may cause a Valsalva maneuver that increases ICP, the nurse should institute a regular bowel program that includes use of a stool softener. For a client at risk for increased ICP, the nurse should prevent, not encourage, oral fluid intake and should elevate the head of the bed only 30 degrees. Suctioning, indicated for a client with lung congestion, isn't necessary for this client.

While performing an initial nursing assessment on a client admitted with suspected tic douloureux (trigeminal neuralgia), for which of the following would the nurse expect to observe?

facial pain in the areas of the 5th cranial nerve Tic douloureux (trigeminal neuralgia) is manifested by pain in the areas of the fifth (trigeminal) cranial nerve. Ptosis and diplopia are associated with myasthenia gravis. Hyporeflexia and weakness of the lower extremities are associated with Guillain-Barre syndrome. Fatigue and depression are associated with multiple sclerosis.

A client with a spinal cord injury says he has difficulty recognizing the symptoms of urinary tract infection (UTI). Which symptom is an early sign of UTI in a client with a spinal cord injury?

fever and change in urine clarity Fever and change in urine clarity as early signs of UTI in a client with a spinal cord injury. Lower back pain is a late sign. A client with a spinal cord injury may not experience a burning sensation or urinary frequency.

A patient has a nursing diagnosis of risk for impaired skin integrity related to immobility and secondary to diabetes. As part of the plan of care, the nurse plans to reposition the patient frequently. Based on an understanding of positioning and its effects, the nurse identifies which position as preferred to the semi-Fowler's position?

recumbent Although a patient should be repositioned laterally, prone, and dorsally in sequence, the recumbent position is preferred to the semi-Fowler's position because this position provides an increased body surface area of support.

A patient had a small pituitary adenoma removed by the transsphenoidal approach and has developed diabetes insipidus. What pharmacologic therapy will the nurse be administering to this patient to control symptoms?

vasopressin Manipulation of the posterior pituitary gland during surgery may produce transient diabetes insipidus of several days' duration (Hickey, 2009). It is treated with vasopressin but occasionally persists.

A client is scheduled for standard EEG testing to evaluate a possible seizure disorder. Which nursing intervention should the nurse perform before the procedure?

withhold anticonvulsant meds for 24 to 48 hours before the exam Anticonvulsant agents, tranquilizers, stimulants, and depressants should be withheld 24 to 48 hours before an EEG because these medications can alter EEG wave patterns or mask the abnormal wave patterns of seizure disorders. To increase the chances of recording seizure activity, it is sometimes recommended that the client be deprived of sleep on the night before the EEG. Coffee, tea, chocolate, and cola drinks are omitted in the meal before the test because of their stimulating effect. However, meals are not omitted, because an altered blood glucose concentration can cause changes in brain wave patterns. The client is informed that a standard EEG takes 45 to 60 minutes; a sleep EEG requires 12 hours.

A patient who has suffered a stroke begins having complications regarding spasticity in the lower extremity. What ordered medication does the nurse administer to help alleviate this problem?

Lioresal (Baclofen) Spasticity, particularly in the hand, can be a disabling complication after stroke. Botulinum toxin type A injected intramuscularly into wrist and finger muscles has been shown to be effective in reducing this spasticity (although the effect is temporary, typically lasting 2 to 4 months) (Teasell, Foley, Pereira, et al., 2012). Other treatments for spasticity may include stretching, splinting, and oral medications such as baclofen (Lioresal).

The nurse in the neurologic ICU is caring for a client who sustained a severe brain injury. Which nursing measures will the nurse implement to help control intracranial pressure (ICP)?

Maintain cerebral perfusion pressure from 50 to 70 mm Hg The nurse should maintain cerebral perfusion pressure from 50 to 70 mm Hg to help control increased ICP. Other measures include elevating the head of the bed as prescribed, maintaining the client's head and neck in neutral alignment (no twisting or flexing the neck), initiating measures to prevent the Valsalva maneuver (e.g., stool softeners), maintaining body temperature within normal limits, administering O2 to maintain PaO2 greater than 90 mm Hg, maintaining fluid balance with normal saline solution, avoiding noxious stimuli (e.g., excessive suctioning, painful procedures), and administering sedation to reduce agitation.

A nurse is reviewing a CT scan of the brain, which states that the client has arterial bleeding with blood accumulation above the dura. Which of the following facts of the disease progression is essential to guide the nursing management of client care?

Monitoring is needed as rapid neurologic deterioration may occur. The nurse identifies that the CT scan suggests an epidural hematoma. A key component in planning care is the understanding that rapid neurologic deterioration occurs. Symptoms evolve quickly. A crash cart may be kept nearby, but this is not the key information. An intracerebral hematoma is bleeding within the brain, which is a different area of bleeding.

The nurse is performing the physical examination of a client with a suspected neurologic disorder. In addition to assessing other parts of the body, the nurse should assess for neck rigidity. Which method should help the nurse assess for neck rigidity correctly?

Moving the head and chin toward the chest The neck is examined for stiffness or abnormal position. The presence of rigidity is assessed by moving the head and chin toward the chest. The nurse should not maneuver the neck if a head or neck injury is suspected or known. The neck should also not be maneuvered if trauma to any part of the body is evident. Moving the head toward the sides or pressing the bones on the neck will not help assess for neck rigidity correctly. While assessing for neck rigidity, sensation at the neck area is not to be assessed.

A client with meningitis has a history of seizures. Which activity should the nurse do while the client is actively seizing?

Turn the client to the side during a seizure and do not restrain movements When a client is in a seizure, the nurse should turn the client to the side and not restrain his or her movements. This helps reduce the potential for aspiration of saliva or stomach contents. The nurse should suction the mouth and pharynx after a seizure has occurred, not during the seizure. Anticonvulsants may be administered to reduce the chances of seizure. Oxygen should not be given to clients with seizures. Clients with respiratory distress are given oxygen. Finally, a cooling blanket is placed beneath the client when hyperthermia occurs, not a seizure.

At a certain point, the brain's ability to autoregulate becomes ineffective and decompensation (ischemia and infarction) begins. Which of the following are associated with Cushing's triad? Select all that apply.

bradypnea hypertension bradycardia The bradycardia, hypertension, and bradypnea associated with this deterioration are known as Cushing's triad, a grave sign. At this point, herniation of the brainstem and occlusion of the cerebral blood flow occur if therapeutic intervention is not initiated immediately.

The critical care nurse is giving end-of-shift report on a client. The nurse uses the Glasgow Coma Scale (GCS) to assess the level of consciousness (LOC) of a female client and reports to the oncoming nurse that the client has an LOC of 6. What does an LOC score of 6 in a client indicate?

comatose The GSC is used to measure the LOC. The scale consists of three parts: eye opening response, best verbal response, and best motor response. A normal response is 15. A score of 7 or less is considered comatose. Therefore, a score of 6 indicates the client is in a state of coma and not in any other state such as stupor or somnolence. The evaluations are recorded on a graphic sheet where connecting lines show an increase or decrease in the LOC.

A client has undergone a lumbar puncture as part of a neurological assessment. The client is put under the care of a nurse after the procedure. Which important postprocedure nursing intervention should be performed to ensure the client's maximum comfort?

encourage the client to drink liberal amounts of fluids The nurse should encourage the client to take liberal fluids and should inspect the injection site for swelling or hematoma. These measures help restore the volume of cerebrospinal fluid extracted. The client is administered antihistamines before a test only if he or she is allergic to contrast dye and contrast dye will be used. The room of the client who has undergone a lumbar puncture should be kept dark and quiet. The client should be encouraged to rest, because sensory stimulation tends to magnify discomfort.

The nurse is caring for a client with aphasia. Which strategy will the nurse use to facilitate communication with the client?

establishing eye contact The following strategies should be used by the nurse to encourage communication with a client with aphasia: face the client and establish eye contact, speak in your usual manner and tone, use short phrases, and pause between phrases to allow the client time to understand what is being said; limit conversation to practical and concrete matters; use gestures, pictures, objects, and writing; and as the client uses and handles an object, say what the object is. It helps to match the words with the object or action. Be consistent in using the same words and gestures each time you give instructions or ask a question, and keep extraneous noises and sounds to a minimum. Too much background noise can distract the client or make it difficult to sort out the message being spoken.

The nurse is working in the rehabilitative setting caring for tetraplegia and paraplegia clients. When instructing family members on the difference between the sites of impairment, which location should the nurse explain differentiates the two disorders?

first thoracic vertebrae Tetraplegia is the impairment of all extremities and the trunk when there is a spinal injury at or above the first thoracic vertebrae. Paraplegia is the impairment of all extremities below the first thoracic vertebrae.

A client experiences loss of consciousness, tongue biting, and incontinence, along with tonic and clonic phases of seizure activity. The nurse should document this episode as which type of seizure?

generalized A generalized seizure causes generalized electrical abnormality in the brain. The client typically falls to the ground, losing consciousness. The body stiffens (tonic phase) and then alternates between episodes of muscle spasm and relaxation (clonic phase). Tongue biting, incontinence, labored breathing, apnea, and cyanosis may also occur. A Jacksonian seizure begins as a localized motor seizure. The client experiences a stiffening or jerking in one extremity, accompanied by a tingling sensation in the same area. Absence seizures occur most commonly in children. They usually begin with a brief change in the level of consciousness, signaled by blinking or rolling of the eyes, a blank stare, and slight mouth movements. Symptoms of a sensory seizure include hallucinations, flashing lights, tingling sensations, vertigo, déjà vu, and smelling a foul odor.

What does the nurse recognize as the earliest sign of serious impairment of brain circulation related to increasing ICP?

lethargy & stupor As ICP increases, the patient becomes stuporous, reacting only to loud or painful stimuli. At this stage, serious impairment of brain circulation is probably taking place, and immediate intervention is required.

The nurse is caring for a patient with Parkinson's disease and is preparing to administer medication. What does the nurse administer to the patient that is considered the most effective drug currently given for the tremor of Parkinson's?

levodopa

A client is about to be discharged after undergoing surgery for the treatment of a brain tumor and has a referral in place for medical and radiation oncology. Which component(s) should be included in the discharge teaching for this client? Select all that apply.

medication regimen appointments for chemo & radio adverse effects of chemo or radiation & techniques for management nutritional support The nurse should include the medication regimen, appointments for chemotherapy and radiotherapy, adverse effects of chemotherapy or radiation and techniques for managing them, and nutritional support as components of the discharge teaching for this client. Electromyography is used in amyotrophic lateral sclerosis (ALS) to validate weakness in the affected muscles and should not be included for the client being discharged after surgery for a brain tumor.

The nurse is assessing a newly admitted client with a diagnosis of meningitis. On assessment, the nurse expects to find which of the following?

positive kernig A positive Kernig's sign is a common finding in the client with meningitis. When the client is lying with the thigh flexed on the abdomen, the leg cannot be completely extended. A positive Brudzinski's sign is usual with meningitis. The Romberg sign would not be tested in this client. The client will develop lethargy as the illness progresses, not hyper-alertness.

A client has sustained a traumatic brain injury with involvement of the hypothalamus. The health care team is concerned about the complication of diabetes insipidus. Which of the following would be an appropriate nursing intervention to monitor for early signs of diabetes insipidus?

record intake & output A record of intake and output is maintained for the client with a traumatic brain injury, especially if the client has hypothalamic involvement and is at risk for the development of diabetes insipidus. Excessive output will alert the nurse to possible fluid imbalance early in the process.

A client who is disoriented and restless after sustaining a concussion during a car accident is admitted to the hospital. Which nursing diagnosis takes the highest priority in this client's care plan?

risk for injury Because the client is disoriented and restless, the most important nursing diagnosis is Risk for injury. Although Disturbed sensory perception (visual), Dressing or grooming self-care deficit, and Impaired verbal communication may all be appropriate, they're secondary because they don't immediately affect the client's health or safety.

A client is receiving hypothermic treatment for uncontrolled fever related to increased intracranial pressure (ICP). Which assessment finding requires immediate intervention?

shivering Shivering can increase intracranial pressure by increasing vasoconstriction and circulating catecholamines. Shivering also increases oxygen consumption. A capillary refill of 2 seconds, urine output of 100mL/hr, and cool, dry skin are expected findings.

A client has a spinal cord injury. The home health nurse is making an initial visit to the client at home and plans on reinforcing teaching on autonomic dysreflexia. What symptom would the nurse stress to the client and his family?

sweating Characteristics of this acute emergency are as follows: severe hypertension; slow heart rate; pounding headache; nausea; blurred vision; flushed skin; sweating; goosebumps (erection of pilomotor muscles in the skin); nasal stuffiness; and anxiety.

apraxia

inability to perform previously learned purposeful motor acts on a voluntary basis

The nurse is providing health education to a client recently diagnosed with a brain tumor. During the appointment, the client states, "I'm really worried that I will have a seizure while I am at work or with my kids. Should I be concerned about this?" How should the nurse respond?

"There is a risk for seizures in people who have this diagnosis. What have you already discussed with your primary health care provider regarding management of seizures?" In this case, the client is verbalizing a valid concern about management of the potential manifestation of the brain tumor. The nurse should engage the client by providing fact-based information about the probability of seizures caused by effects of brain tumors. The nurse should further engage in the discussion by evaluating the client's existing understanding of the seizures related to brain tumors and the associated management of this problem. The open-ended manner in which the nurse has asked the question in the correct answer option allows the client to reveal any knowledge deficits or gaps in understanding of the condition. Telling the client there is a strong chance that he or she will have a seizure is countertherapeutic and would serve to increase the client's anxiety. The nurse's aim should be to reduce the client's anxiety related to the diagnosis. Telling the client that seizures are a genetic neurological condition is out of context in this situation. The client is worried about having a seizure because he or she has a brain tumor. The nurse should address the concern in the correct context. The nurse is incorrect when stating having this discussion is not within the nurse's scope of practice. The client's verbalized concern presents an opportunity for the nurse to evaluate the client's understanding of the treatment and management of the condition. The nurse should refer the client back to the primary health care provider if there are any aspects of the client's health history that are unclear.

A client with a traumatic brain injury has already displayed early signs of increasing intracranial pressure (ICP). Which of the following would be considered late signs of increasing ICP?

Decerebrate posturing and loss of corneal reflex Early indications of increasing ICP include disorientation, restlessness, increased respiratory effort, mental confusion, pupillary changes, weakness on onside of the body or in one extremity, and constant, worsening headache. Later indications of increasing ICP include decreasing level of consciousness until client is comatose, decreased or erratic pulse and respiratory rate, increased blood pressure and temperature, widened pulse pressure, Cheyne-Stokes breathing, projectile vomiting, hemiplegia or decorticate or decerebrate posturing, and loss of brain stem reflexes (pupillary, corneal, gag, and swallowing).

traumatic brain injury

an injury to the skull or brain that is severe enough to interfere with normal funciton

An older client complains of a constant headache. A physical examination shows papilledema. What may the symptoms indicate in this client?

brain tumor The incidence of brain tumor increases with age. Headache and papilledema are less common symptoms of a brain tumor in the older adult. Symptoms of epilepsy include fits and spasms, while symptoms of trigeminal neuralgia would be pain in the jaws or facial muscles. Hypostatic pneumonia develops due to immobility or prolonged bed rest in older clients.

To evaluate a client's cerebellar function, a nurse should ask:

do you have any problems with balance To evaluate cerebellar function, the nurse should ask the client about problems with balance and coordination. The nurse asks about difficulty speaking or swallowing to assess the functions of cranial nerves IX, X, and XII. Questions about muscle strength help her evaluate the client's motor system.

A client is demonstrating an altered level of consciousness from a traumatic brain injury. Which assessment will the nurse use as a sensitive indicator of neurologic function?

glasgow coma scale An altered level of consciousness (LOC) is present when the client is not oriented, does not follow commands, or needs persistent stimuli to achieve a state of alertness. LOC is gauged on a continuum, with a normal state of alertness and full cognition (consciousness) on one end and coma on the other end. LOC, a sensitive indicator of neurologic function, is assessed based on the criteria in the Glasgow Coma Scale: eye opening, verbal response, and motor response. Cerebellar function, cranial nerve function, and mental status evaluation are all elements of the neurologic assessment.

receptive aphasia

inability to understand what someone is saying (damage to temporal lobe area)

myringotomy

incision in the tympanic membrane

rigidity

increase in muscle tone at rest, characterized by increased resistance to passive stretch

Which term describes the fibrous connective tissues that cover the brain and spinal cord?

meninges The meninges have three layers: the dura mater, arachnoid mater, and pia mater. The dura mater is the outermost layer of the protective covering of the brain and spinal cord. The arachnoid is the middle membrane, and the pia mater is the innermost membrane of this protective covering.

A patient is brought to the emergency department with a possible stroke. What initial diagnostic test for a stroke, usually performed in the emergency department, would the nurse prepare the patient for?

noncontrast computed tomogram The initial diagnostic test for a stroke is usually a noncontrast computed tomography (CT) scan. This should be performed within 25 minutes or less from the time the patient presents to the emergency department (ED) to determine if the event is ischemic or hemorrhagic (the category of stroke determines treatment).

Which method is used to help reduce intracranial pressure?

using a cervical collar

dysphonia

voixe impairment of altered voice production

paresthesia

abnormal sensation of numbness and tingling

bradykinesia

abnormally slow voluntary movements and speech

Myasthenia gravis occurs when antibodies attack which receptor sites?

acetylcholine In myasthenia gravis, antibodies directed at the acetylcholine receptor sites impair transmission of impulses across the myoneural junction. Serotonin, dopamine, and gamma-aminobutyric acid are not receptor sites that are attacked in myasthenia gravis.

Which occurs when reflexes are hyperactive when the foot is abruptly dorsiflexed?

clonus Clonus occurs when the foot is abruptly dorsiflexed. It continues to "beat" two or three times before it settles into a position of rest. Sustained clonus always indicates the present of central nervous system disease and requires further evaluation. Ataxia is incoordination of voluntary muscle action. Rigidity is an increase in muscle tone at rest characterized by increased resistance to passive movement. Flaccid posturing is usually the result of lower brain stem dysfunction; the client has no motor function, is limp, and lacks motor tone.

Which is the most common cause of acute encephalitis in the United States?

herpes simplex virus HSV-1 ( herpes simplex virus) is the most common cause of acute encephalitis in the United States. Fungal infections of the central nervous system occur rarely in healthy people. The Western equine encephalitis virus is one of four types of arboviral encephalitis that occur in North America is one of several fungi that may cause fungal encephalitis. Lyme disease leads to flu like symptoms and starts as a local infection which can systematically spread causing organ issues, however the incidence is rate, HIV leads to autoimmune disorders.

The nurse is educating a patient with a seizure disorder. What nutritional approach for seizure management would be beneficial for this patient?

high in protein low in carb A dietary intervention, referred to as the ketogenic diet, may be helpful for control of seizures in some patients. This high-protein, low-carbohydrate, high-fat diet is most effective in children whose seizures have not been controlled with two antiseizure medications, but it is sometimes used for adults who have had poor seizure control

A 64-year-old client reports symptoms consistent with a transient ischemic attack (TIA) to the health care provider in the emergency department. Which is the origin of the client's symptoms?

impaired cerebral circulation TIAs involve the same mechanism as in the ischemic cascade, but symptoms are transient (< 24 hours) and there is no evidence of cerebral tissue infarction. The ischemic cascade begins when cerebral blood flow decreases to less than 25 mL/100 g/min and neurons are no longer able to maintain aerobic respiration. Thus, a TIA results directly from impaired blood circulation in the brain. Atherosclerosis, cardiac disease, hypertension, or diabetes can be risk factors for a TIA but do not cause it.

A nurse is performing passive range of motion to a client's upper extremities. The nurse touches the client's thumb to each fingertip on the same hand. The nurse is performing which of the following?

opposition Opposition involves touching the thumb to each fingertip on the same hand. Adduction would involve moving the arm away from the midline of the body. Pronation involves rotating the forearm so that the palm of the hand is down. Dorsiflexion involves movement that flexes or bends the hand back toward the body.

The critical care nurse is caring for a client with bacterial meningitis. The client has developed cerebral vasculitis and increased ICP. What neurologic sequelae might this client develop?

optic Neurologic sequelae in survivors include damage to the cranial nerves that facilitate vision and hearing. Sequelae to meningitis do not include damage to the vagal nerve, the olfactory nerve or the facial nerve.

A client with a T4-level spinal cord injury (SCI) reports severe headache. The nurse notes profuse diaphoresis of the client's forehead and scalp and suspects autonomic dysreflexia. What is the first thing the nurse will do?

place the client in a sitting position The nurse immediately places the client in a sitting position to lower blood pressure. Next, the nurse will do a rapid assessment to identify and alleviate the cause, and then check the bladder and bowel. The nurse will examine skin for any places of irritation. If no cause can be found, the nurse will give an antihypertensive as ordered and continue to look for cause. He or she watches for rebound hypotension once cause is alleviated. Antiembolic stockings will not decrease the blood pressure.

A nurse working in the neurologic intensive care unit admits from the emergency department a patient with an inoperable brain tumor. Upon entering the room, the nurse observes that the patient is positioned like part B of the accompanying image. Based on this initial observation, what would the nurse predict about this patient's prognosis?

poor An inappropriate or nonpurposeful response is random and aimless. Posturing may be decorticate or decerebrate. Decerebrate posturing indicates deeper and more severe dysfunction than does decorticate posturing; it implies brain pathology, which is a poor prognostic sign. Decorticate posture is the flexion and internal rotation of forearms and hands. Decerebrate posture is extension and external rotation. Flaccidity is the absence of motor response and the most severe neurologic impairment.

The nurse is participating in a health fair for stroke prevention. Which will the nurse say is a modifiable risk factor for ischemic stroke?

smoking Modifiable risk factors for transient ischemic attack (TIA) and ischemic stroke include hypertension, type 1 diabetes, cardiac disease, smoking, and chronic alcoholism. Advanced age, gender, and race are nonmodifiable risk factors for stroke.

Which term refers to muscular hypertonicity in a weak muscle, with increased resistance to stretch?

spasticity Spasticity is often associated with weakness, increased deep tendon reflexes, and diminished superficial reflexes. Akathisia refers to restlessness, an urgent need to move around, and agitation. Ataxia refers to impaired ability to coordinate movement. Myoclonus refers to spasm of a single muscle or group of muscles.

Guillain-Barré syndrome is an autoimmune attack on the peripheral myelin sheath. Which of the following is an action of myelin?

speeds nerve impulse transmission Myelin is a complex substance that covers nerves, providing insulation and speeding the conduction of impulses from the cell body to the dendrites. The axon carries the message to the next nerve cell. The neuron is the building block of the nervous system. A neurotransmitter is a chemical messenger.

A nurse in a rehabilitation facility is coordinating the discharge of a client who is tetraplegic. The client, who is married and has two children in high school, is being discharged to home and will require much assistance. Who would the discharge planner recognize as being the most important member of this client's care team?

spouse The client's spouse and family would need to be involved in the everyday care of the client; without their support, it is unlikely that the client would be able to manage at home.

A client is admitted to the hospital after sustaining a closed head injury in a skiing accident. The physician ordered neurologic assessments to be performed every 2 hours. The client's neurologic assessments have been unchanged since admission, and the client is complaining of a headache. Which intervention by the nurse is best?

Assess the client's neurologic status for subtle changes, administer acetaminophen, and then reassess the client in 30 minutes. Headache is common after a head injury. Therefore, the nurse should administer acetaminophen to try to manage the client's pain without causing sedation. The nurse should then reassess the client in 30 minutes to note the effectiveness of the pain medication. Administering codeine, an opioid, could cause sedation that may mask changes in the client's neurologic status. Although a headache is expected, the client should receive treatment to alleviate pain. The nurse should notify the physician if the client's neurologic status changes or if treatment doesn't relieve the headache.

Which interventions are appropriate for a client with increased intracranial pressure (ICP)? Select all that apply.

Maintaining aseptic technique with an intraventricular catheter Administering prescribed antipyretics Frequent oral care Controlling fever is an important intervention for a client with increased ICP because fevers can cause an increase in cerebral metabolism and can lead to cerebral edema. Antipyretics are appropriate to control a fever. It is imperative that the nurse use aseptic technique when caring for the intraventricular catheter because of its risk for infection. Oral care should be provided frequently because the client is likely to be placed on a fluid restriction and will have dry mucous membranes. A nondrying oral rinse may be used. Coughing should be discouraged in a client with increased ICP because it increases intrathoracic pressure, and thus ICP. Unless contraindicated, the head of the bed should be elevated to 30 to 45 degrees and in a neutral position to allow for venous drainage.

A client reports light-headedness, speech disturbance, and left-sided weakness lasting for several hours. The neurologist diagnosed a transient ischemic attack, which caused the client great concern. What would the nurse include during client education?

When symptoms cease, the client will return to presymptomatic state. Impaired blood circulation can be caused by arteriosclerosis, cardiac disease, or diabetes. A TIA is a sudden, brief episode of neurologic impairment. Symptoms may disappear within 1 hour; some continue for as long as 1 day. One third of people who experience a TIA subsequently develop a stroke.

A nurse is teaching a community group about modifiable and nonmodifiable risk factors for ischemic strokes. Which of the following is a risk factor that cannot be modified?

advanced age Modifiable risk factors for ischemic stroke include hypertension, atrial fibrillation, hyperlipidemia, diabetes mellitus, smoking, asymptomatic carotid stenosis, obesity, and excessive alcohol consumption. Advanced age is a nonmodifiable risk factor.

The nurse enters the client's room and finds the client with an altered level of consciousness (LOC). Which is the nurse's priority concern?

airway clearance The most important consideration in managing the patient with altered LOC is to establish an adequate airway and ensure ventilation.

Which is a late sign of increased intracranial pressure (ICP)?

altered respiratory patterns Altered respiratory patterns are late signs of increased ICP and may indicate pressure or damage to the brainstem. Headache, irritability, and any change in LOC are early signs of increased ICP. Speech changes, such as slowed speech or slurring, are also early signs of increased ICP.

spinal cord injury

an injury to the spinal cord vertebral column supporting soft tissue,, or intervertebral discs caused by trauma

secondary injury

an insult to the brain subsequent to the original traumatic event

Which term refers to the inability to perform previously learned purposeful motor acts on a voluntary basis?

apraxia

penumbra regions

area of low cerebral blood flow

epilepsy

at least two unprovoked seizures occurring more than 24 hours apart

Which drug should be available to counteract the effect of edrophonium chloride?

atropine Atropine should be available to control the side effects of edrophonium chloride. Prednisone, azathioprine, and pyridostigmine bromide are not used to counteract these effects.

A patient recently noted difficulty maintaining his balance and controlling fine movements. The nurse explains that the provider will order diagnostic studies for the part of his brain known as the:

cerbellum The cerebellum is largely responsible for coordination of all movement. It also controls fine movement, balance, position (postural) sense or proprioception (awareness of where each part of the body is), and integration of sensory input.

Which of the following outcomes would be most appropriate to include in the plan of care for a client diagnosed with a muscular dystrophy?

client participates in activities of daily living using adaptive devices The muscular dystrophies are a group of incurable muscle disorders characterized by progressive weakening and wasting of the skeletal or voluntary muscles. Nursing care focuses on maintaining the client at his or her optimal level of functioning and enhancing the quality of life. Therefore, the outcome of participating in activities of daily living with adaptive devices would be most appropriate. Medications are not used to treat these disorders; however, they may be necessary if the client develops a complication such as respiratory dysfunction. The disorder is incurable and progressive, not chronic. Diagnostic follow-up would provide little if any information about the course of the disorder.

A 45-year-old client presents to the ED reporting trouble speaking and numbness of the right arm and leg. The nurse suspects an ischemic stroke. Which insult or abnormality can cause an ischemic stroke?

cocaine Two classifications of ischemic strokes are cryptogenic strokes, which have no known cause, and strokes from other causes, such as illicit drug use, coagulopathies, migraine, and spontaneous dissection of the carotid or vertebral arteries. Cocaine is a potent vasoconstrictor and may result in a life-threatening reaction, even with the individual's first use of the drug. Arteriovenous malformations, trauma, and intracerebral aneurysm are associated with hemorrhagic strokes.

A nurse is working in the neurologic intensive care unit and admits from the emergency department a patient with an inoperable brain tumor. Upon entering the room, the nurse observes that the patient is positioned like the person in part B of the accompanying image. Which posturing is the patient exhibiting?

decerebrate An inappropriate or nonpurposeful response is random and aimless. Posturing may be decorticate or decerebrate. Decorticate posture is the flexion and internal rotation of forearms and hands. Decerebrate posture is extension and external rotation. Flaccidity is the absence of motor response; tonic clonic movements are seen with seizures.

The nurse is completing an assessment on a client with myasthenia gravis. Which of the following historical recounting provides the most significant evidence regarding when the disorder began?

drooping eyelids Ptosis (eyelid drooping) is the most common manifestation of myasthenia gravis. Muscle weakness varies depending on the muscles affected. Shortness of breath and respiratory distress occurs later as the disease progresses. Muscle spasms are more likely in multiple sclerosis. Photophobia is not significant in myasthenia gravis.

A nurse is providing care to a client who has had a stroke. Which symptoms are consistent with left-sided stroke?

impulsive behavior, poor judgment, deficits in left visual fields Impulsive behavior, poor judgment, deficits in left visual fields are symptoms of right hemispheric stroke. Expressive aphasia, defects in the right visual fields, problems with abstract thinking are symptoms of left hemispheric stroke. Problems with abstract thinking, impairment of short-term memory, poor judgment are symptoms inconsistent with each other as some indicate left and others indicate right hemispheric stroke. Cautious behavior, deficits in left visual fields, misjudgment of distances are symptoms inconsistent with each other as some indicate left and others indicate right hemispheric stroke.

The nurse is caring for a patient postoperatively after intracranial surgery for the treatment of a subdural hematoma. The nurse observes an increase in the patient's blood pressure from the baseline and a decrease in the heart rate from 86 to 54. The patient has crackles in the bases of the lungs. What does the nurse suspect is occurring?

increased icp Increased ICP and bleeding are life threatening to the patient who has undergone intracranial surgery. An increase in blood pressure and decrease in pulse with respiratory failure may indicate increased ICP.

The nurse is seeing a female client who has been diagnosed with a pituitary adenoma. During the clinic visit, the client tells the nurse that she has been having irregular menstrual periods despite having very regular menstrual periods all her life. The nurse knows this physiological change is likely related to which characteristic of this type of brain tumor?

increased prolactin

The nurse is caring for a client with Guillain-Barré syndrome. Which assessment finding would indicate the need for oral suctioning?

increased pulse, adventitious breath sounds An increased pulse rate above baseline with adventitious breath sounds indicate compromised respirations and signal a need for airway clearance. A decrease in pulse rate is not indicative of airway obstruction. An increase of pulse rate with slight elevation of respirations (16 breaths/minute) is not significant for suctioning unless findings suggest otherwise.

A client is receiving an IV infusion of mannitol (Osmitrol) after undergoing intracranial surgery to remove a brain tumor. To confirm that this drug is producing its therapeutic effect, the nurse should consider which finding most significant?

increased urine output The therapeutic effect of mannitol is diuresis, which is confirmed by an increased urine output. A decreased LOC and elevated blood pressure may indicate lack of therapeutic effectiveness. A decreased heart rate doesn't indicate that mannitol is effective.

A client has been diagnosed with a frontal lobe brain abscess. Which nursing intervention is appropriate?

initiate seizure precautions A frontal lobe brain abscess produces seizures, hemiparesis, and frontal headache; therefore, the nurse should anticipate the need for seizure precautions. Facial weakness and visual disturbances are associated with a temporal lobe abscess. The client may experience expressive aphasia related to the abscess, but that does not indicate the need to ensure the client takes in nothing by mouth.

A nurse is preparing to administer an antiseizure medication to a client. Which of the following is an appropriate antiseizure medication?

lamictal Lamictal is an antiseizure medication. Its packaging was recently changed in an attempt to reduce medication errors, because this medication has been confused with Lamisil (an antifungal), labetalol (an antihypertensive), and Lomotil (an antidiarrheal).

A 53-year-old man presents to the emergency department with a chief complaint of inability to form words, and numbness and weakness of the right arm and leg. Where would you locate the site of injury?

left frontoparietal region The patient is exhibiting signs of expressive aphasia with numbness/tingling and weakness of the right arm and leg. This indicates injury to the expressive speech center (Broca's area), which is located in the inferior portion of the frontal lobe. The motor strip is located in the posterior portion of the frontal lobe. The sensory strip is located in the anterior parietal lobe.

A client was undergoing conservative treatment for a herniated nucleus pulposus, at L5 - S1, which was diagnosed by magnetic resonance imaging. Because of increasing neurologic symptoms, the client undergoes lumbar laminectomy. The nurse should take which step during the immediate postoperative period?

logroll client from side to side Logrolling the client maintains alignment of his hips and shoulders and eliminates twisting in his operative area. The nurse should encourage ROM exercises to maintain muscle strength. Because of pressure on the operative area, having the client sit up in a chair or with the head of the bed elevated should be allowed only for short durations.

The nurse is called to attend to a patient having a seizure in the waiting area. What nursing care is provided for a patient who is experiencing a convulsive seizure? Select all that apply.

loosening constrictive clothing providing for privacy positioning the patient on his or her side with head flexed forward During a patient's seizure, the nurse should do the following. Loosen constrictive clothing. If possible, place the patient on one side with head flexed forward, which allows the tongue to fall forward and facilitates drainage of saliva and mucus. If suction is available, use it if necessary to clear secretions. Provide privacy, and protect the patient from curious onlookers. (The patient who has an aura [warning of an impending seizure] may have time to seek a safe, private place.) The nurse should not attempt to pry open jaws that are clenched in a spasm or attempt to insert anything. Broken teeth and injury to the lips and tongue may result from such an action. No attempt should be made to restrain the patient during the seizure, because muscular contractions are strong and restraint can produce injury.

The nurse is discussing spinal cord injury (SCI) at a health fair at a local high school. The nurse relays that the most common cause of SCI is

motor vehicle crashes The most common causes of SCIs are motor vehicle crashes (46%), falls (22%), violence (16%), and sports (12%). Males account for 80% of clients with SCI. An estimated 50% to 70% of SCIs occur in those aged 15 to 35 years.

The nurse collects neurologic data and determines that the client has significant visual deficits. A brain tumor is considered. Which area of the brain does the nurse consider to be most likely to contain the neurologic deficit?

occipital The visual receiving area is in the occipital lobe; therefore, this is the area of the brain the nurse determines is affected for the client with significant visual deficits. The frontal lobe contains the written and motor speech areas. The parietal lobe is the primary sensory area of the brain. The temporal lobe is the auditory receiving and association area of the brain, and is responsible for speech comprehension (i.e., Wernicke area).

A client with neurologic infection develops cerebral edema from syndrome of inappropriate antidiuretic hormone (SIADH). Which is an important nursing action for this client?

restricting fluid intake & hydration Fluid restriction may be necessary if the client develops cerebral edema and hypervolemia from SIADH. Antipyretics are administered to clients who develop hyperthermia. In addition, it is important to maintain adequate hydration in such clients. A client with neurologic infection should be given tracheal suctioning and hyperoxygenation only when the respiratory distress develops.

The nurse is caring for a client with Bell's palsy. Which body system will the nurse identify as the priority for this client?

sensory The priority for nursing care of the client with Bell's palsy is protecting the eye from injury. The eyelid often will not close completely and the blink reflex is diminished, increasing the risk of injury from dust and foreign particles. Corneal irritation and ulcerations may occur. The condition is caused by unilateral inflammation of the 7th cranial nerve; however, the neurologic system is not at risk for additional injury or effects. The integumentary system is not affected by Bell's palsy. The facial muscles on the affected side will be paralyzed; however, this is the only area of the musculoskeletal status affected.

Which neurotransmitter demonstrates inhibitory action, helps control mood and sleep, and inhibits pain pathways?

serotonin The brain stem, hypothalamus, and dorsal horn of the spinal cord are sources of serotonin. Enkephalin is excitatory and associated with pleasurable sensations. Norepinephrine is usually excitatory and affects mood and overall activity. Acetylcholine is usually excitatory, but the parasympathetic effects are sometimes inhibitory.

A nurse is working in an outpatient studies unit administering neurological tests. The client is surprised that paste is used to secure an electroencephalogram and asks how it will be removed from the hair. With what substance does the nurse reply?

shampoo Shampoo is used to remove the paste, which attached the electrodes to the head. Acetone is not used on the hair. There is no special soap needed. More than warm water is needed to lift and remove the paste.

A nurse is caring for a 16-year-old adolescent with a head injury resulting from a fight after a high school football game. A physician has intubated the client and written orders to wean him from sedation therapy. A nurse needs further assessment data to determine whether:

she'll have to apply restraints to prevent the client from dislodging the endotracheal (ET) tube. When the client isn't sedated, he may make attempts to remove the ET tube without realizing what he's doing. The nurse needs to obtain information to determine whether it's necessary to request an order for restraints. The nurse doesn't need to obtain additional data to determine if the nutritional protocol will continue to reflect the client's needs because this aspect of care won't change. The client doesn't require additional assessments to continue I.V. administration of medications. I.V. medication clearly needs to continue because the client is intubated. The staff nurse doesn't need to monitor payment status because client sedation shouldn't affect payment status.

The nursing is assessing a client who has been diagnosed with a pituitary adenoma, but has not yet started treatment. The client reports having increased heart rate, hand tremors, difficulty sleeping, weight loss and hyperthermia. The nurse anticipates the client will require blood work to assess for overproduction of which hormone?

thyroid-stimulating hormone In clients diagnosed with pituitary tumors, increase may be seen in prolactin hormone, growth hormone, adrenocorticotropic hormone, or thyroid-stimulating hormone. In this case, the client is exhibiting symptoms related to hyperthyroidism and the blood work should include the thyroid-stimulating hormone level to determine if an overproduction of this hormone due to the presence of the tumor is the cause of the presenting symptoms.

A client who was found unconscious at home is brought to the hospital by a rescue squad. In the intensive care unit, the nurse checks the client's oculocephalic (doll's eye) response by:

turning the client's head suddenly while holding the eyelids open. To elicit the oculocephalic response, which detects cranial nerve compression, the nurse turns the client's head suddenly while holding the eyelids open. Normally, the eyes move from side to side when the head is turned; in an abnormal response, the eyes remain fixed. The nurse introduces ice water into the external auditory canal when testing the oculovestibular response; normally, the client's eyes deviate to the side of ice water introduction. The nurse touches the client's cornea with a wisp of cotton to elicit the corneal reflex response, which reveals brain stem function; blinking is the normal response. Shining a bright light into the client's pupil helps evaluate brain stem and cranial nerve III function; normally, the client's pupil responds by constricting.

Which disease includes loss of motor neurons in the anterior horns of the spinal cord and motor nuclei of the lower brain stem?

amyotrophic lateral sclerosis Amyotrophic lateral sclerosis (ALS) is a disease of unknown cause in which there is a loss of motor neurons in the anterior horns of the spinal cord and the motor nuclei of the lower brain stem. Parkinson disease is a slowly progressing neurologic movement disorder that eventually leads to disability. Alzheimer disease is a chronic, progressive, and degenerative brain disorder that is accompanied by profound effects on memory, cognition and ability for self-care. Huntington disease is a chronic, progressive, hereditary disease of the nervous system that results in progressive involuntary choreiform movement and dementia.

The nurse is caring for a client who is being assessed for brain death. Which are cardinal signs of brain death? Select all that apply.

apnea coma absence of brainstem reflexes The three cardinal signs of brain death on clinical examination are coma, the absence of brain stem reflexes, and apnea. Adjunctive tests, such as cerebral blood flow studies, electroencephalography, transcranial Doppler, and brain stem auditory evoked potential, are often used to confirm brain death.

Which term refers to the inability to coordinate muscle movements, resulting in difficulty walking?

ataxia Ataxia is the inability to coordinate voluntary muscle action; tremors (rhythmic, involuntary movements) noted at rest or during movement suggest a problem in the anatomic areas responsible for balance and coordination. Agnosia is the loss of ability to recognize objects through a particular sensory system. Spasticity is the sustained increase in tension of a muscle when it is passively lengthened or stretched.

During a Tensilon test to determine if a patient has myasthenia gravis, the patient complains of cramping and becomes diaphoretic. Vital signs are BP 130/78, HR 42, and respiration 18. What intervention should the nurse prepare to do?

atropine to control the side effects of the edrophonium Atropine should be available to control the side effects of edrophonium, which include bradycardia, sweating, and cramping.

Lesions in the temporal lobe may result in which type of agnosia?

auditory Lesions in the temporal lobe (lateral and superior portions) may result in auditory agnosia. Lesions in the occipital lobe may result in visual agnosia. Lesions in the parietal lobe may result in tactile agnosia. Lesions in the parietal lobe (posteroinferior regions) may result in relationship and body part agnosia.

You are a neurotrauma nurse working in a neuro ICU. What would you know is an acute emergency and is seen in clients with a cervical or high thoracic spinal cord injury after the spinal shock subsides?

autonomic dysreflexia Autonomic dysreflexia is an acute emergency and is seen in clients with a cervical or high thoracic spinal cord injury, usually after the spinal shock subsides. Tetraplegia results in the paralysis of all extremities when there is a high cervical spine injury. Paraplegia occurs with injuries at the thoracic level. Areflexia is a loss of sympathetic reflex activity below the level of injury within 30 to 60 minutes of a spinal injury.

A client with a T4 level spinal cord injury (SCI) is complaining of a severe headache. The nurse notes profuse diaphoresis of the client's forehead and scalp. Which of the following does the nurse suspect?

autonomic dysreflexia Autonomic dysreflexia occurs only after spinal shock has resolved. It is characterized by a severe, pounding headache, marked hypertension, diaphoresis, nausea, nasal congestion, and bradycardia. It occurs only with SCIs above T6 and is a hypertensive emergency. It is not related to thrombophlebitis.

A client reports light-headedness, speech disturbance, and left-sided weakness that have lasted for several hours. In the examination, an abnormal sound is auscultated in an artery leading to the brain. What is the term for the auscultated discovery?

bruit A neurologic examination during an attack reveals neurologic deficits. Auscultation of the artery may reveal a bruit (abnormal sound caused by blood flowing over a rough surface within one or both carotid arteries). The term for the auscultated discovery is "bruit."

When assessing a client who has experienced a spinal injury, the nurse notes diaphragmatic breathing and loss of upper limb use and sensation. At what level does the nurse anticipate the injury has occurred?

c5 At level C5, the client retains full head and neck control. At C1 the client has little or no sensation or control of the head and neck. At C2 to C3 the client feels head and neck sensation and has some neck control. At C4 the client has good head and neck sensation and motor control.

At which of the following spinal cord injury levels does the patient have full head and neck control?

c5 At the level of C5, the patient should have full head and neck control, shoulder strength, and elbow flexion. At C4 injury, the patient will have good head and neck sensation and motor control, some shoulder elevation, and diaphragm movement. At C2 to C3, the patient will have head and neck sensation, some neck control, and can be independent of mechanical ventilation for short periods of time.

The nurse is caring for a client who underwent surgery to remove a spinal cord tumor. When conducting the postoperative assessment, the nurse notes the presence of a bulge at the surgical site. The nurse suspects the client is experiencing what complication from the surgery?

cerebrospinal fluid leakage Bulging at the incision may indicate a contained cerebrospinal fluid (CSF) leak. The site should be monitored for increasing bulging, known as pseudomeningocele, which may require surgical repair. Infection at the surgical site should be suspected if the surgical dressing is stained. The bulge does not indicate growth of secondary tumor, this can only be identified using diagnostic imaging. Impaired tissue healing would be indicated if the nurse assessed redness, swelling and warmth at the surgical site during a dressing change. The bulge at the site warrants further assessment of a postsurgical leak of CSF.

A nurse observes that decerebrate posturing is a comatose client's response to painful stimuli. Decerebrate posturing as a response to pain indicates:

dysfunction in the brain stem. Decerebrate posturing indicates damage of the upper brain stem. Decorticate posturing indicates cerebral dysfunction. Increased intracranial pressure is a cause of decortication and decerebration. Alterations in sensation or paralysis indicate dysfunction in the spinal column.

A client undergoes a craniotomy with supratentorial surgery to remove a brain tumor. On the first postoperative day, the nurse notes the absence of a bone flap at the operative site. How should the nurse position the client's head?

elevated 30 degrees After supratentorial surgery, the nurse should elevate the client's head 30 degrees to promote venous outflow through the jugular veins. The nurse would keep the client's head flat after infratentorial, not supratentorial, surgery. However, after supratentorial surgery to remove a chronic subdural hematoma, the neurosurgeon may order the nurse to keep the client's head flat; typically, the client with such a hematoma is older and has a less expandable brain. A client without a bone flap can't be positioned with the head turned onto the operative side because doing so may injure brain tissue. Elevating the head 10 degrees or less wouldn't promote venous outflow through the jugular veins.

A client in the emergency department has a suspected neurologic disorder. To assess gait, the nurse asks the client to take a few steps; with each step, the client's feet make a half circle. To document the client's gait, the nurse should use which term?

helicopod A helicopod gait is an abnormal gait in which the client's feet make a half circle with each step. An ataxic gait is staggering and unsteady. In a dystrophic gait, the client waddles with the legs far apart. In a steppage gait, the feet and toes rise high off the floor and the heel comes down heavily with each step.

The nurse practitioner advises a patient who is at high risk for a stroke to be vigilant in his medication regimen, to maintain a healthy weight, and to adopt a reasonable exercise program. This advice is based on research data that shows the most important risk factor for stroke is:

hypertension Hypertension is the most modifiable risk factor for either ischemic or hemorrhagic stroke. Unfortunately, it remains under-recognized and undertreated in most communities.

A client is admitted to an acute care facility with a suspected dysfunction of the lower brain stem. The nurse should monitor this client closely for:

hypoxia Lower brain stem dysfunction alters bulbar functions, such as breathing, talking, swallowing, and coughing. Therefore, the nurse should monitor the client closely for hypoxia. Temperature control, vision, and gait aren't lower brain stem functions.

A patient with a brain tumor is complaining of headaches that are worse in the morning. What does the nurse know could be the reason for the morning headaches?

increased intracranial pressure Headache, although not always present, is most common in the early morning and is made worse by coughing, straining, or sudden movement. It is thought to be caused by the tumor invading, compressing, or distorting the pain-sensitive structures or by edema that accompanies the tumor, leading to increased intracranial pressure.

The nurse is caring for a client with a head injury. The client is experiencing CSF rhinorrhea. Which order should the nurse question?

insertion of an NG tube Clients with brain injury are assumed to be catabolic, and nutritional support consultation should be considered as soon as the client is admitted. Parenteral nutrition via a central line or enteral feedings administered via an NG or nasojejunal feeding tube should be considered. If cerebrospinal fluid rhinorrhea occurs, an oral feeding tube should be inserted instead of a nasal tube. Serial studies of blood and urine electrolytes and osmolality are done because head injuries may be accompanied by disorders of sodium regulation. Urine is tested regularly for acetone. An intervention to maintain skin integrity is getting the client out of bed to a chair three times daily.

A client spends most of his time in a wheelchair. The nurse would be especially alert for the development of pressure ulcers in which area?

ischial tuberosity For a client who sits for prolonged periods, such as in a wheelchair, the ischial tuberosity would be highly susceptible to pressure ulcer development. Areas such as the greater trochanter and lateral malleolus would be susceptible for clients lying on their side. The scapula would be considered a high risk area for clients lying on their back.

An older adult experienced a cerebrovascular disease 6 weeks ago and is currently receiving inpatient rehabilitation. The nurse is coaching the client to contract and relax her muscles while keeping her extremity in a fixed position. Which type of exercise is the client performing?

isometric Isometric exercises are those in which there is alternating contraction and relaxation of a muscle while keeping the part in a fixed position. This exercise is performed by the client. Passive exercises are carried out by the therapist or the nurse without assistance from the client. Resistive exercises are carried out by the client working against resistance produced by either manual or mechanical means. Abduction is movement of a part away from the midline of the body.

The nurse educator is teaching nursing students about various types of brain tumors. The instructor recognizes that teaching has been effective when students correctly identify a client whose lab work indicates excessively high levels of thyroid stimulating hormone would most likely be diagnosed with which type of tumor?

pituitary adenoma Pituitary adenomas can increased production of several hormones including TSH, ACTH, growth hormone and prolactin. Excessive hormone production is not characteristic of the brain tumors identified in the alternate options.

The nurse is evaluating the serum albumin of a client newly admitted on the rehabilitation unit. The nurse determines that the client's serum albumin concentration is low, indicating that the client has which deficiency?

protein Serum albumin is a sensitive indicator of protein deficiency. Serum albumin is not an indicator of potassium, calcium, or phosphorous deficiency.

Which is a component of the nursing management of the client with new variant Creutzfeldt-Jakob disease (vCJD)?

providing supportive care vCJD is a progressive fatal disease, and no treatment is available. Because of the fatal outcome of vCJD, nursing care is primarily supportive and palliative. Prevention of disease transmission is an important part of providing nursing care. Although client isolation is not necessary, use of standard precautions is important. Institutional protocols are followed for blood and body fluid exposure and decontamination of equipment. Organ donation is not an option because of the risk for disease transmission. Amphotericin B is used in the treatment of fungal encephalitis; no treatment is available for vCJD.

The nurse working on a neurological unit is mentoring a nursing student who asks about a client who has sustained primary and secondary brain injuries. The nurse correctly tells the student which of the following, related to the secondary injury?

results from inadequate delivery of nutrients and oxygen to the cells Secondary injury results from inadequate delivery of nutrients and oxygen to the cells, usually as a result of cerebral edema and increased intracranial pressure. Primary injury results from initial damage related to the traumatic event.

While riding a bicycle in a race, a patient fell into a ditch and sustained a head injury. Another cyclist found the patient lying unconscious in the ditch and called 911. What type of concussion does the patient most likely have?

3 There are three grades of concussion or mild traumatic brain injury defined by the American Academy of Neurology when the injury is sports related (Ruff, Iverson, Barth, et al., 2009). A grade 1 concussion has symptoms of transient confusion, no loss of consciousness, and duration of mental status abnormalities on examination that resolve in less than 15 minutes. A grade 2 concussion also has symptoms of transient confusion and no loss of consciousness, but the concussion symptoms or mental status abnormalities on examination last more than 15 minutes. In a grade 3 concussion, there is any loss of consciousness lasting from seconds to minutes (Ruff et al., 2009).

A 154-pound woman has been prescribed tPA (0.9 mg/kg) for an ischemic stroke. The nurse knows to give how many mg initially?

6.3 A person who weighs 154 lbs weighs 70 kg. To calculate total dosage, multiply 70 × 0.9 mg/kg = 63 mg. Ten percent of the calculated dose is given as an IV bolus over 1 minute. The remaining dose (90%) is given IV over 1 hour via an infusion pump. So initially the nurse gives 10% (6.3 mg) over 1 minute.

The ED nurse is receiving a client handoff report at the beginning of the nursing shift. The departing nurse notes that the client with a head injury shows Battle sign. The incoming nurse expects which to observe clinical manifestation?

An area of bruising over the mastoid bone Fractures of the base of the skull tend to traverse the paranasal sinus of the frontal bone or the middle ear located in the temporal bone Therefore, they frequently produce hemorrhage from the nose, pharynx, or ears, and blood may appear under the conjunctiva. An area of ecchymosis (bruising) may be seen over the mastoid (Battle sign). Basilar skull fractures are suspected when cerebrospinal fluid (CSF) escapes from the ears (CSF otorrhea) and the nose (CSF rhinorrhea). Drainage of CSF is a serious problem because meningeal infection can occur if organisms gain access to the cranial contents via the nose, ear, or sinus through a tear in the dura. A bloodstain surrounded by a yellowish stain on the head dressing is referred to as a halo sign and is highly suggestive of a CSF leak.

A client with tetraplegia cannot do his own skin care. The nurse is teaching the caregiver about the importance of maintaining skin integrity. Which of the following will the nurse most encourage the caregiver to do?

Maintain a diet for the client that is high in protein, vitamins, and calories. During the period immediately following a spinal cord injury, spinal shock occurs. In spinal shock, all reflexes are absent and the extremities are flaccid. When spinal shock subsides, the client will demonstrate positive Babinski's reflex, hyperreflexia, and spasticity of all four extremities.

The nurse is performing an initial nursing assessment on a client with possible Guillain-Barre syndrome. Which of the following findings would be most consistent with this diagnosis?

Muscle weakness and hyporeflexia of the lower extremities Guillain-Barre syndrome typically begins with muscle weakness and diminished reflexes of the lower extremities. Fever, skin rash, cough, and ptosis are not signs/symptoms associated with Guillain-Barre.

For a client who has had a stroke, which nursing intervention can help prevent contractures in the lower legs?

attaching braces or splints to each foot and leg Attaching braces or splints to each foot and leg prevents foot drop (a lower leg contracture) by supporting the feet in proper alignment. Putting slippers on the client's feet can't prevent foot drop because slippers are too soft to support the ankle joints. Crossing the ankles every 2 hours is contraindicated because it can cause excess pressure and damage veins, promoting thrombus formation. Placing hand rolls on the balls of each foot doesn't prevent contractures because hand rolls are too soft to support and hold the feet in proper alignment.

A nurse is reading a journal article about stroke and the underlying causes associated with this condition. The nurse demonstrates understanding of the information when identifying which subtype of stroke as being due to atrial fibrillation?

cardio embolic Ischemic strokes are further divided into five subtypes, according to a mechanism-based classification system: large-artery thrombotic strokes (representing 20% of ischemic strokes); small, penetrating artery thrombotic strokes (25%); cardio embolic strokes (20%); cryptogenic strokes (strokes that cannot be attributed to any specific cause) (30%); and "other" (5%). Large-artery thrombotic strokes are caused by atherosclerotic plaques in the large blood vessels of the brain. Thrombus formation and occlusion can occur at the site of the atherosclerosis and result in ischemia and infarction (tissue death). Small, penetrating artery thrombotic strokes that affect one or more vessels and cause reduced blood flow are the most common type of ischemic stroke, typically caused by longstanding hypertension, hyperlipidemia, or diabetes. Cardio embolic strokes are associated with cardiac dysrhythmias, such as atrial fibrillation, but can also be associated with valvular heart disease or left ventricular thrombus. The last two classifications of ischemic strokes are cryptogenic strokes, which have no identified cause, and strokes from other causes, such as illicit drug use (cocaine), coagulopathies, migraine, or spontaneous dissection of the carotid or vertebral arteries.

A client comes to the clinic for evaluation because of complaints of dizziness and difficulty walking. Further assessment reveals a staggering gait, marked muscle incoordination, and nystagmus. A brain tumor is suspected. Based on the client's assessment findings, the nurse would suspect that the tumor is located in which area of the brain?

cerebellum Findings such as ataxic or staggering gait, dizziness, marked muscle incoordination, and nystagmus suggest a cerebellar tumor. A frontal lobe tumor frequently produces personality, emotional, and behavioral changes. A tumor in the motor cortex produces seizure-like movements localized on one side of the body. Occipital lobe tumors produce visual manifestations.

The office nurse is reviewing an 80-year-old female client's reports related to the onset of a severe headache, rated at 9 out of 10 on the pain scale, with recent onset. The client denies any visual changes. During a prior visit to the office a few months ago, the client had reported a ground-level fall as a result of falling off a chair and hitting the back of their head. The client had been taken to the emergency department, where imaging was performed with negative results. The nurse anticipates that the client has developed ____________________________________ and that ____________________________________________ will be ordered

chronic subdural hematoma, CT imaging of the brain This client has had a prior head trauma with a negative imaging scan. Prior head trauma can lead to the development of a chronic subdural hematoma, which presents with symptoms such as severe headache, mental deterioration, focal neurologic changes, personality changes, and/or symptoms that the client is having a stroke. There is no indication that the client had follow-up imaging based on the prior head trauma, which should be included in protocol management of head injuries. Prior head trauma can lead to the development of a chronic subdural hematoma. Based on the clinical presentation of a severe headache, this is the most likely clinical diagnosis. Based on the clinical presentation, follow-up imaging is indicated to confirm the presence of a chronic subdural hematoma, which can occur following a recent head trauma. Because the head trauma occurred a few months ago, an acute finding would have presented earlier, at the time of injury. The differential diagnosis of chronic subdural hematoma includes a stroke but there is insufficient clinical evidence to support this finding. An electrocardiogram (ECG) is not indicated at this time because there is no provided clinical evidence of any cardiac abnormalities. Coagulation studies are not indicated at this time because the priority is to obtain an imaging study.

While snowboarding, a client fell and sustained a blow to the head, resulting in a loss of consciousness. The client regained consciousness within an hour after arrival at the ED, was admitted for 24-hour observation, and was discharged without neurologic impairment. What would the nurse expect this client's diagnosis to be?

concussion A concussion results from a blow to the head that jars the brain. It usually is a consequence of falling, striking the head against a hard surface such as a windshield, colliding with another person (e.g., between athletes), battering during boxing, or being a victim of violence. The force of the blow causes temporary neurologic impairment but no serious damage to cerebral tissue. There is generally complete recovery within a short time.

When the nurse observes that the patient has extension and external rotation of the arms and wrists, and extension, plantar flexion, and internal rotation of the feet, she records the patient's posturing as which of the following?

decerebrate Decerebrate posturing is the result of lesions at the midbrain and is more ominous than decorticate posturing. The described posturing results from cerebral trauma and is not normal. The patient has no motor function, is limp, and lacks motor tone with flaccid posturing. In decorticate posturing, the patient has flexion and internal rotation of the arms and wrists and extension, internal rotation, and plantar flexion of the feet.

A nurse is instructing the spouse of a client who suffered a stroke about the use of eating devices the client will be using. During the teaching, the spouse starts to cry and states, "One minute he is laughing, and the next he's crying; I just don't understand what's wrong with him." Which statement is the best response by the nurse?

emotional lability is common after a stroke and it usually improves over time This is the most therapeutic and informative response. Often, most relatives of clients with stroke handle the physical changes better than the emotional aspects of care. The family should be prepared to expect occasional episodes of emotional lability. The client may laugh or cry easily and may be irritable and demanding or depressed and confused. The nurse can explain to the family that the client's laughter does not necessarily connote happiness, nor does crying reflect sadness, and that emotional lability usually improves with time. The remaining responses are nontherapeutic and do not address the spouse's concerns.

Which term refers to the shifting of brain tissue from an area of high pressure to an area of low pressure?

herniation With a herniation, the herniated tissue exerts pressure on the brain area into which it has shifted, which interferes with the blood supply in that area. Cessation of cerebral blood flow results in cerebral ischemia, infarction, and brain death. Autoregulation is an ability of cerebral blood vessels to dilate or constrict to maintain stable cerebral blood flow despite changes in systemic arterial blood pressure. Cushing response is the brain's attempt to restore blood flow by increasing arterial pressure to overcome the increased ICP. The Monro-Kellie hypothesis is a theory that states that because of limited space for expansion within the skull, an increase in any one of the cranial contents causes a change in the volume of the others.

The nursing instructor gives their students an assignment of making a plan of care for a client with Huntington's disease. What would be important for the students to include in the teaching portion of the care plan?

how to facilitate tasks such as using both hands t hold a drinking glass The nurse demonstrates how to facilitate tasks such as using both hands to hold a drinking glass, using a straw to drink, and wearing slip-on shoes. The teaching portion of the care plan would not include how to exercise, perform household tasks, or take a bath.

The nurse is caring for a client admitted with a stroke. Imaging studies indicate an embolus partially obstructing the right carotid artery. What type of stroke does the nurse know this client has?

ischemic Ischemic strokes occur when a thrombus or embolus obstructs an artery carrying blood to the brain; about 80% of strokes are the ischemic variety. The other options are incorrect.

The nurse is providing postoperative care for a client who just underwent surgery to remove a metastatic intramedullary tumor. On postoperative day 3, the client states, "I am really looking forward to going running again, it had become too difficult because of the loss of feeling in my feet." Which should the nurse address in the client's care plan?

knowledge deficit Clients with extensive neurologic deficits before surgery usually do not make significant functional recovery, even after successful tumor removal. In this case, the client had already developed bilateral sensory loss in the lower extremities indicating the fairly progressed impact of the tumor on the client's functional ability. The client's statement reflects a knowledge deficit and it is a priority to provide information regarding the possibility that lower extremity sensation may not return. Although body image disturbance and anxiety may be identified and addressed. This would occur after the client demonstrates an accurate understanding of loss of functional capabilities as a result of the progressed tumor. Ensuring the client understands the extent of functional loss due to the impact of the tumor is a priority. The client does not demonstrate impaired cognition.

The nurse is caring for a client diagnosed with a hemorrhagic stroke. The nurse recognizes that which intervention is most important?

maintaining a patent airway Maintaining the airway is the most important nursing intervention. Immediate complications of a hemorrhagic stroke include cerebral hypoxia, decreased cerebral blood flow, and extension of the area of injury. Providing adequate oxygenation of blood to the brain minimizes cerebral hypoxia. Brain function depends on delivery of oxygen to the tissues. Administering supplemental oxygen and maintaining hemoglobin and hematocrit at acceptable levels will assist in maintaining tissue oxygenation. All other interventions are appropriate, but the airway takes priority. The head of the bed should be elevated to 30 degrees, monitoring the client because of the risk for seizures, and stool softeners are recommended to prevent constipation and straining, but these are not the most important interventions.

A nurse is reviewing the medical record of a client diagnosed with a primary brain tumor. The nurse identifies the type of tumor as one that is most commonly found in adults. Which type of tumor would the nurse most likely identify?

meningioma Although acoustic neuromas, pituitary adenomas and angiomas can occur in adults, meningiomas are the most common type of brain tumor found in adults.

A nurse is assessing a patient's urinary output as an indicator of diabetes insipidus. The nurse knows that an hourly output of what volume over 2 hours may be a positive indicator?

more than 200 mL/hr For patients undergoing dehydrating procedures, vital signs, including blood pressure, must be monitored to assess fluid volume status. An indwelling urinary catheter is inserted to permit assessment of renal function and fluid status. During the acute phase, urine output is monitored hourly. An output greater than 200 mL per hour for 2 consecutive hours may indicate the onset of diabetes insipidus

The nurse is caring for a comatose client. The nurse knows she should assess the client's motor response. Which method may the nurse use to assess the motor response?

observing the clients response to painful stimulus The nurse evaluates motor response in a comatose or unconscious client by administering a painful stimulus. This action helps determine if the client makes an appropriate response by reaching toward or withdrawing from the stimulus. The Romberg test is used to assess equilibrium in a noncomatose client. Pupils are examined for their reaction to light to assess sensitivity in the third cranial (oculomotor) nerve. Sensitivity to temperature, touch, and pain is a test to assess the sensory function of the client and not motor response.

A nurse is performing a baseline assessment of a client's skin integrity. What are the priority assessments? Select all that apply.

overall risk of developing pressure ulcers potential areas of pressure ulcer development presence of pressure ulcers on the client When assessing skin integrity, the overall risk potential of developing pressure ulcers takes priority. Overall risk assessment encompasses review of existing pressure ulcers as well as potential areas for development of pressure ulcers. Foley catheter output and family history of pressure ulcers are not important when assessing skin integrity.

Which of the following is a late symptom of spinal cord compression?

paralysis Later symptoms include evidence of motor weakness and sensory deficits progressing to paralysis. Early symptoms associated with spinal cord compression include bladder and bowel dysfunction (urinary incontinence or retention; fecal incontinence or constipation).

A client presents to the emergency department stating numbness and tingling occurring down the left leg into the left foot. When documenting the experience, which medical terminology would the nurse be most correct to report?

paresthesia When a client reports numbness and tingling in an area, the client is reporting a paresthesia. The nurse would document the experience as such or place the client's words in parentheses. The nurse would not make a medical diagnosis of sciatic nerve pain or herniation. The symptoms are not consistent with paralysis.

During a neurological assessment examination, the nurse assesses a patient for tactile agnosia. The nurse places a familiar door key in the patient's hand and asks him to identify the object with his eyes closed. The nurse documents his inability to identify the object and notes the affected area of the brain. Which of the following is the most likely affected area of the brain?

parietal The parietal lobe analyzes sensor information and relays the interpretation to the cortical area. Failure to identify a familiar object by touch is indicative of parietal lobe dysfunction.

Which lobe of the brain is responsible for spatial relationships?

parietal The parietal lobe is essential to a person's awareness of body position in space, size and shape discrimination, and right-left orientation. The frontal lobe controls information storage or memory and motor function. The temporal lobe contains the auditory receptive area. The occipital lobe is responsible for visual interpretation.

A patient has difficulty interpreting his awareness of body position in space. Which lobe is most likely to be damaged?

parietal The parietal lobe is the primary sensory cortex. It is essential to a person's awareness of his body in space, as well as orientation in space and spatial relations.

Which disease is associated with decreased levels of dopamine due to destruction of pigmented neuronal cells?

parkinson In some clients, Parkinson disease can be controlled; however, it cannot be cured. Multiple sclerosis is a chronic, degenerative, progressive disease of the central nervous system (CNS) characterized by the occurrence of small patches of demyelination in the brain and spinal cord. Huntington disease is a chronic, progressive, hereditary disease of the nervous system that results in progressive involuntary dancelike movements and dementia. Creutzfeldt-Jakob disease is a rare, transmissible, progressive and fatal disease of the CNS characterized by spongiform degeneration of the gray matter of the brain.

Which of the following drugs may be used after a seizure to maintain a seizure-free state?

phenobarbital IV diazepam (Valium), lorazepam (Ativan), or fosphenytoin (Cerebyx) are administered slowly in an attempt to halt seizures immediately. Other medications (phenytoin, phenobarbital) are administered later to maintain a seizure-free state. In general, a single drug is used to control the seizures.

Autonomic dysreflexia is an acute emergency that occurs with spinal cord injury as a result of exaggerated autonomic responses to stimuli. Which of the following is the initial nursing intervention to treat this condition?

raise the head of bed and place patient in a sitting position The head of the bed is raised and the patient is placed immediately in a sitting position to lower blood pressure. Assessment of body systems is done after the emergency has been addressed.

A client with a T4-level spinal cord injury (SCI) is experiencing autonomic dysreflexia; his blood pressure is 230/110. The nurse cannot locate the cause and administers antihypertensive medication as ordered. The nurse empties the client's bladder and the symptoms abate. Now, what must the nurse watch for?

rebound hypotension When the cause is removed and the symptoms abate, the blood pressure goes down. The antihypertensive medication is still working. The nurse must watch for rebound hypotension. Rebound hypertension is not an issue. Spinal shock occurs right after the initial injury. The client is not at any more risk for a urinary tract infection after the episode than he was before.

A nurse is caring for a client who requires a wheelchair. Which piece of equipment impedes circulation to the area it's meant to protect?

ring or donut The nurse shouldn't use rings or donuts with any client because this equipment restricts circulation. Specialty mattresses evenly distribute pressure. Gel pads redistribute the client's weight, and water beds distribute pressure over the entire surface.

During assessment, a patient reports that she sometimes "wets herself" when sneezing. The nurse documents this as which of the following?

stress incontinence Stress incontinence occurs when perineal muscles weaken. Urine subsequently leaks when the intra-abdominal pressure increases, such as with sneezing or coughing. Urge incontinence refers to the involuntary elimination of urine associated with a strong perceived need to void. Functional incontinence occurs in patients with intact urinary physiology but who experience mobility impairment, environmental barriers, or cognitive problems and cannot reach and use the toilet before soiling themselves. Reflex or neurogenic incontinence is associated with a spinal cord lesion that interrupts cerebral control, resulting in no sensory awareness of the need to void.

A patient with Bell's palsy says to the nurse, "It doesn't hurt anymore to touch my face. How am I going to get muscle tone back so I don't look like this anymore?" What interventions can the nurse suggest to the patient?

suggest massaging the face several times daily, using a gentle upward motion to maintain muscle tone After the sensitivity of the nerve to touch decreases and the patient can tolerate touching the face, the nurse can suggest massaging the face several times daily, using a gentle upward motion, to maintain muscle tone. Facial exercises, such as wrinkling the forehead, blowing out the cheeks, and whistling, may be performed with the aid of a mirror to prevent muscle atrophy. Exposure of the face to cold and drafts is avoided.

The nurse is instructing a community class when a student asks, "How does someone get super strength in an emergency?" The nurse should respond by describing the action of the:

sympathetic The division of the autonomic nervous system called the sympathetic nervous system regulates the expenditure of energy. The neurotransmitters of the sympathetic nervous system are called catecholamines. During an emergency situation or an intensely stressful event, the body adjusts to deliver blood flow and oxygen to the brain, muscles, and lungs that need to react in the situation. The musculoskeletal system benefits from the sympathetic nervous system as the fight-or-flight effects pump blood to the muscles. The parasympathetic nervous system works to conserve body energy not expend it during an emergency. The endocrine system regulates metabolic processes.

A nurse is evaluating a stage II pressure ulcer on a client. Which wound assessment findings should prompt the nurse to request a referral from the wound care nurse?

wound measuring 2 cm × 2 cm × 0.5 cm with tan leathery appearance A wound (regardless of its size) that contains tan, leathery tissue requires evaluation by the wound care nurse. This wound most likely requires debridement before wound healing can take place. Although option A describes a large wound, it's showing signs of healing, so a consult isn't necessary. Option B describes a stage II wound that has a clean wound bed; a wound nurse consult isn't necessary for this type of wound. The wound described in option D is small and shows signs of healing; a wound care consult isn't required at this time.


Related study sets

Structure and Function of Plasma Membranes

View Set

Chapter 6: When Governments Intervene in Markets

View Set

Kinns Chapter 17 Banking Services and Procedures

View Set

Chapter 1: The Earliest Human Societies

View Set

Management of Patients with Chronic Pulmonary Diseases

View Set